Универсальный внешний накопитель для всех iOS-устройств, совместим с PC/Mac, Android
Header Banner
8 800 100 5771 | +7 495 540 4266
c 9:00 до 24:00 пн-пт | c 10:00 до 18:00 сб
0 Comments

Содержание

Все главные формулы по физике – Физика – Теория, тесты, формулы и задачи

Оглавление:

 

Кинематика

К оглавлению…

Путь при равномерном движении:

Перемещение S (расстояние по прямой между начальной и конечной точкой движения) обычно находится из геометрических соображений. Координата при равномерном прямолинейном движении изменяется по закону (аналогичные уравнения получаются для остальных координатных осей):

Средняя скорость пути:

Средняя скорость перемещения:

Определение ускорения при равноускоренном движении:

Выразив из формулы выше конечную скорость, получаем более распространённый вид предыдущей формулы, которая теперь выражает зависимость скорости от времени при равноускоренном движении:

Средняя скорость при равноускоренном движении:

Перемещение при равноускоренном прямолинейном движении может быть рассчитано по нескольким формулам:

Координата при равноускоренном движении

изменяется по закону:

Проекция скорости при равноускоренном движении изменяется по такому закону:

Скорость, с которой упадет тело падающее с высоты h без начальной скорости:

Время падения тела с высоты h без начальной скорости:

Максимальная высота на которую поднимется тело, брошенное вертикально вверх с начальной скоростью v0, время подъема этого тела на максимальную высоту, и полное время полета (до возвращения в исходную точку):

Формула для тормозного пути тела:

Время падения тела при горизонтальном броске с высоты H может быть найдено по формуле:

Дальность полета тела при горизонтальном броске с высоты H:

Полная скорость в произвольный момент времени при горизонтальном броске, и угол наклона скорости к горизонту:

Максимальная высота подъема при броске под углом к горизонту (относительно начального уровня):

Время подъема до максимальной высоты при броске под углом к горизонту:

Дальность полета и полное время полета тела брошенного под углом к горизонту (при условии, что полет заканчивается на той же высоте с которой начался, т.е. тело бросали, например, с земли на землю):

Определение периода вращения при равномерном движении по окружности:

Определение частоты вращения при равномерном движении по окружности:

Связь периода и частоты:

Линейная скорость при равномерном движении по окружности может быть найдена по формулам:

Угловая скорость вращения при равномерном движении по окружности:

Связь линейной и скорости и угловой скорости выражается формулой:

Связь угла поворота и пути при равномерном движении по окружности радиусом R (фактически, это просто формула для длины дуги из геометрии):

Центростремительное ускорение находится по одной из формул:

 

Динамика

К оглавлению…

Второй закон Ньютона:

Здесь: F – равнодействующая сила, которая равна сумме всех сил действующих на тело:

Второй закон Ньютона в проекциях на оси (именно такая форма записи чаще всего и применяется на практике):

Третий закон Ньютона (сила действия равна силе противодействия):

Сила упругости:

Общий коэффициент жесткости параллельно соединённых пружин:

Общий коэффициент жесткости последовательно соединённых пружин:

Сила трения скольжения (или максимальное значение силы трения покоя):

Закон всемирного тяготения:

Если рассмотреть тело на поверхности планеты и ввести следующее обозначение:

Где: g – ускорение свободного падения на поверхности данной планеты, то получим следующую формулу для силы тяжести:

Ускорение свободного падения на некоторой высоте от поверхности планеты выражается формулой:

Скорость спутника на круговой орбите:

Первая космическая скорость:

Закон Кеплера для периодов обращения двух тел вращающихся вокруг одного притягивающего центра:

 

Статика

К оглавлению…

Момент силы определяется с помощью следующей формулы:

Условие при котором тело не будет вращаться:

Координата центра тяжести системы тел (аналогичные уравнения для остальных осей):

 

Гидростатика

К оглавлению…

Определение давления задаётся следующей формулой:

Давление, которое создает столб жидкости находится по формуле:

Но часто нужно учитывать еще и атмосферное давление, тогда формула для общего давления на некоторой глубине

h в жидкости приобретает вид:

Идеальный гидравлический пресс:

Любой гидравлический пресс:

КПД для неидеального гидравлического пресса:

Сила Архимеда (выталкивающая сила, V – объем погруженной части тела):

 

Импульс

К оглавлению…

Импульс тела находится по следующей формуле:

Изменение импульса тела или системы тел (обратите внимание, что разность конечного и начального импульсов векторная):

Общий импульс системы тел (важно то, что сумма векторная):

Второй закон Ньютона в импульсной форме может быть записан в виде следующей формулы:

Закон сохранения импульса. Как следует из предыдущей формулы, в случае если на систему тел не действует внешних сил, либо действие внешних сил скомпенсировано (равнодействующая сила равна нолю), то изменение импульса равно нолю, что означает, что общий импульс системы сохраняется:

Если внешние силы не действуют только вдоль одной из осей, то сохраняется проекция импульса на данную ось, например:

 

Работа, мощность, энергия

К оглавлению…

Механическая работа рассчитывается по следующей формуле:

Самая общая формула для мощности (если мощность переменная, то по следующей формуле рассчитывается средняя мощность):

Мгновенная механическая мощность:

Коэффициент полезного действия (КПД) может быть рассчитан и через мощности и через работы:

Формула для кинетической энергии:

Потенциальная энергия тела поднятого на высоту:

Потенциальная энергия растянутой (или сжатой) пружины:

Полная механическая энергия:

Связь полной механической энергии тела или системы тел и работы внешних сил:

Закон сохранения механической энергии (далее – ЗСЭ). Как следует из предыдущей формулы, если внешние силы не совершают работы над телом (или системой тел), то его (их) общая полная механическая энергия остается постоянной, при этом энергия может перетекать из одного вида в другой (из кинетической в потенциальную или наоборот):

 

Молекулярная физика

К оглавлению…

Химическое количество вещества находится по одной из формул:

Масса одной молекулы вещества может быть найдена по следующей формуле:

Связь массы, плотности и объёма:

Основное уравнение молекулярно-кинетической теории (МКТ) идеального газа:

Определение концентрации задаётся следующей формулой:

Для средней квадратичной скорости молекул имеется две формулы:

Средняя кинетическая энергия поступательного движения одной молекулы:

Постоянная Больцмана, постоянная Авогадро и универсальная газовая постоянная связаны следующим образом:

Следствия из основного уравнения МКТ:

Уравнение состояния идеального газа (уравнение Клапейрона-Менделеева):

Газовые законы. Закон Бойля-Мариотта:

Закон Гей-Люссака:

Закон Шарля:

Универсальный газовый закон (Клапейрона):

Давление смеси газов (закон Дальтона):

Тепловое расширение тел. Тепловое расширение газов описывается законом Гей-Люссака. Тепловое расширение жидкостей подчиняется следующему закону:

Для расширения твердых тел применяются три формулы, описывающие изменение линейных размеров, площади и объема тела:

 

Термодинамика

К оглавлению…

Количество теплоты (энергии) необходимое для нагревания некоторого тела (или количество теплоты выделяющееся при остывании тела) рассчитывается по формуле:

Теплоемкость (

С – большое) тела может быть рассчитана через удельную теплоёмкость (c – маленькое) вещества и массу тела по следующей формуле:

Тогда формула для количества теплоты необходимой для нагревания тела, либо выделившейся при остывании тела может быть переписана следующим образом:

Фазовые превращения. При парообразовании поглощается, а при конденсации выделяется количество теплоты равное:

При плавлении поглощается, а при кристаллизации выделяется количество теплоты равное:

При сгорании топлива выделяется количество теплоты равное:

Уравнение теплового баланса (ЗСЭ). Для замкнутой системы тел выполняется следующее (сумма отданных теплот равна сумме полученных):

Если все теплоты записывать с учетом знака, где «+» соответствует получению энергии телом, а «–» выделению, то данное уравнение можно записать в виде:

Работа идеального газа:

Если же давление газа меняется, то работу газа считают, как площадь фигуры под графиком в pV координатах. Внутренняя энергия идеального одноатомного газа:

Изменение внутренней энергии рассчитывается по формуле:

Первый закон (первое начало) термодинамики (ЗСЭ):

Для различных изопроцессов можно выписать формулы по которым могут быть рассчитаны полученная теплота Q, изменение внутренней энергии ΔU и работа газа A. Изохорный процесс (V = const):

Изобарный процесс (p = const):

Изотермический процесс (T = const):

Адиабатный процесс (Q = 0):

КПД тепловой машины может быть рассчитан по формуле:

Где: Q1 – количество теплоты полученное рабочим телом за один цикл от нагревателя, Q2 – количество теплоты переданное рабочим телом за один цикл холодильнику. Работа совершенная тепловой машиной за один цикл:

Наибольший КПД при заданных температурах нагревателя T1 и холодильника T2, достигается если тепловая машина работает по циклу Карно. Этот КПД цикла Карно равен:

Абсолютная влажность рассчитывается как плотность водяных паров (из уравнения Клапейрона-Менделеева выражается отношение массы к объему и получается следующая формула):

Относительная влажность воздуха может быть рассчитана по следующим формулам:

Потенциальная энергия поверхности жидкости площадью S:

Сила поверхностного натяжения, действующая на участок границы жидкости длиной L:

Высота столба жидкости в капилляре:

При полном смачивании θ = 0°, cos θ = 1. В этом случае высота столба жидкости в капилляре станет равной:

При полном несмачивании θ = 180°, cos θ = –1 и, следовательно, h < 0. Уровень несмачивающей жидкости в капилляре опускается ниже уровня жидкости в сосуде, в которую опущен капилляр.

 

Электростатика

К оглавлению…

Электрический заряд может быть найден по формуле:

Линейная плотность заряда:

Поверхностная плотность заряда:

Объёмная плотность заряда:

Закон Кулона (сила электростатического взаимодействия двух электрических зарядов):

Где: k – некоторый постоянный электростатический коэффициент, который определяется следующим образом:

Напряжённость электрического поля находится по формуле (хотя чаще эту формулу используют для нахождения силы действующей на заряд в данном электрическом поле):

Принцип суперпозиции для электрических полей (результирующее электрическое поле равно векторной сумме электрических полей составляющих его):

Напряженность электрического поля, которую создает заряд Q на расстоянии r от своего центра:

Напряженность электрического поля, которую создает заряженная плоскость:

Потенциальная энергия взаимодействия двух электрических зарядов выражается формулой:

Электрическое напряжение это просто разность потенциалов, т.е. определение электрического напряжения может быть задано формулой:

В однородном электрическом поле существует связь между напряженностью поля и напряжением:

Работа электрического поля может быть вычислена как разность начальной и конечной потенциальной энергии системы зарядов:

Работа электрического поля в общем случае может быть вычислена также и по одной из формул:

В однородном поле при перемещении заряда вдоль его силовых линий работа поля может быть также рассчитана по следующей формуле:

Определение потенциала задаётся выражением:

Потенциал, который создает точечный заряд или заряженная сфера:

Принцип суперпозиции для электрического потенциала (результирующий потенциал равен скалярной сумме потенциалов полей составляющих итоговое поле):

Для диэлектрической проницаемости вещества верно следующее:

Определение электрической ёмкости задаётся формулой:

Ёмкость плоского конденсатора:

Заряд конденсатора:

Напряжённость электрического поля внутри плоского конденсатора:

Сила притяжения пластин плоского конденсатора:

Энергия конденсатора (вообще говоря, это энергия электрического поля внутри конденсатора):

Объёмная плотность энергии электрического поля:

 

Электрический ток

К оглавлению…

Сила тока может быть найдена с помощью формулы:

Плотность тока:

Сопротивление проводника:

Зависимость сопротивления проводника от температуры задаётся следующей формулой:

Закон Ома (выражает зависимость силы тока от электрического напряжения и сопротивления):

Закономерности последовательного соединения:

Закономерности параллельного соединения:

Электродвижущая сила источника тока (ЭДС) определяется с помощью следующей формулы:

Закон Ома для полной цепи:

Падение напряжения во внешней цепи при этом равно (его еще называют напряжением на клеммах источника):

Сила тока короткого замыкания:

Работа электрического тока (закон Джоуля-Ленца). Работа А электрического тока протекающего по проводнику обладающему сопротивлением преобразуется в теплоту Q выделяющуюся на проводнике:

Мощность электрического тока:

Энергобаланс замкнутой цепи

Полезная мощность или мощность, выделяемая во внешней цепи:

Максимально возможная полезная мощность источника достигается, если R = r и равна:

Если при подключении к одному и тому же источнику тока разных сопротивлений R1 и R2 на них выделяются равные мощности то внутреннее сопротивление этого источника тока может быть найдено по формуле:

Мощность потерь или мощность внутри источника тока:

Полная мощность, развиваемая источником тока:

КПД источника тока:

Электролиз

Масса m вещества, выделившегося на электроде, прямо пропорциональна заряду Q, прошедшему через электролит:

Величину k называют электрохимическим эквивалентом. Он может быть рассчитан по формуле:

Где: n – валентность вещества, NA – постоянная Авогадро, M – молярная масса вещества, е – элементарный заряд. Иногда также вводят следующее обозначение для постоянной Фарадея:

 

Магнетизм

К оглавлению…

Сила Ампера, действующая на проводник с током помещённый в однородное магнитное поле, рассчитывается по формуле:

Момент сил действующих на рамку с током:

Сила Лоренца, действующая на заряженную частицу движущуюся в однородном магнитном поле, рассчитывается по формуле:

Радиус траектории полета заряженной частицы в магнитном поле:

Модуль индукции B магнитного поля прямолинейного проводника с током I на расстоянии R от него выражается соотношением:

Индукция поля в центре витка с током радиусом R:

Внутри соленоида длиной l и с количеством витков N создается однородное магнитное поле с индукцией:

Магнитная проницаемость вещества выражается следующим образом:

Магнитным потоком Φ через площадь S контура называют величину заданную формулой:

ЭДС индукции рассчитывается по формуле:

При движении проводника длиной l в магнитном поле B со скоростью v также возникает ЭДС индукции (проводник движется в направлении перпендикулярном самому себе):

Максимальное значение ЭДС индукции в контуре состоящем из N витков, площадью S, вращающемся с угловой скоростью ω в магнитном поле с индукцией В:

Индуктивность катушки:

Где: n – концентрация витков на единицу длины катушки:

Связь индуктивности катушки, силы тока протекающего через неё и собственного магнитного потока пронизывающего её, задаётся формулой:

ЭДС самоиндукции возникающая в катушке:

Энергия катушки (вообще говоря, это энергия магнитного поля внутри катушки):

Объемная плотность энергии магнитного поля:

 

Колебания

К оглавлению…

Уравнение описывающее физические системы способные совершать гармонические колебания с циклической частотой ω0:

Решение предыдущего уравнения является уравнением движения для гармонических колебаний и имеет вид:

Период колебаний вычисляется по формуле:

Частота колебаний:

Циклическая частота колебаний:

Зависимость скорости от времени при гармонических механических колебаниях выражается следующей формулой:

Максимальное значение скорости при гармонических механических колебаниях:

Зависимость ускорения от времени при гармонических механических колебаниях:

Максимальное значение ускорения при механических гармонических колебаниях:

Циклическая частота колебаний математического маятника рассчитывается по формуле:

Период колебаний математического маятника:

Циклическая частота колебаний пружинного маятника:

Период колебаний пружинного маятника:

Максимальное значение кинетической энергии при механических гармонических колебаниях задаётся формулой:

Максимальное значение потенциальной энергии при механических гармонических колебаниях пружинного маятника:

Взаимосвязь энергетических характеристик механического колебательного процесса:

Энергетические характеристики и их взаимосвязь при колебаниях в электрическом контуре:

Период гармонических колебаний в электрическом колебательном контуре определяется по формуле:

Циклическая частота колебаний в электрическом колебательном контуре:

Зависимость заряда на конденсаторе от времени при колебаниях в электрическом контуре описывается законом:

Зависимость электрического тока протекающего через катушку индуктивности от времени при колебаниях в электрическом контуре:

Зависимость напряжения на конденсаторе от времени при колебаниях в электрическом контуре:

Максимальное значение силы тока при гармонических колебаниях в электрическом контуре может быть рассчитано по формуле:

Максимальное значение напряжения на конденсаторе при гармонических колебаниях в электрическом контуре:

Переменный ток характеризуется действующими значениями силы тока и напряжения, которые связаны с амплитудными значениями соответствующих величин следующим образом. Действующее значение силы тока:

Действующее значение напряжения:

Мощность в цепи переменного тока:

Трансформатор

Если напряжение на входе в трансформатор равно U1, а на выходе U2, при этом число витков в первичной обмотке равно n1, а во вторичной n2, то выполняется следующее соотношение:

Коэффициент трансформации вычисляется по формуле:

Если трансформатор идеальный, то выполняется следующее соотношение (мощности на входе и выходе равны):

В неидеальном трансформаторе вводится понятие КПД:

Волны

Длина волны может быть рассчитана по формуле:

Разность фаз колебаний двух точек волны, расстояние между которыми l:

Скорость электромагнитной волны (в т.ч. света) в некоторой среде:

Скорость электромагнитной волны (в т.ч. света) в вакууме постоянна и равна с = 3∙108 м/с, она также может быть вычислена по формуле:

Скорости электромагнитной волны (в т.ч. света) в среде и в вакууме также связаны между собой формулой:

При этом показатель преломления некоторого вещества можно рассчитать используя формулу:

 

Оптика

К оглавлению…

Оптическая длина пути определяется формулой:

Оптическая разность хода двух лучей:

Условие интерференционного максимума:

Условие интерференционного минимума:

Формула дифракционной решетки:

Закон преломления света на границе двух прозрачных сред:

Постоянную величину n21 называют относительным показателем преломления второй среды относительно первой. Если n1 > n2, то возможно явление полного внутреннего отражения, при этом:

Формула тонкой линзы:

Линейным увеличением линзы Γ называют отношение линейных размеров изображения и предмета:

 

Атомная и ядерная физика

К оглавлению…

Энергия кванта электромагнитной волны (в т.ч. света) или, другими словами, энергия фотона вычисляется по формуле:

Импульс фотона:

Формула Эйнштейна для внешнего фотоэффекта (ЗСЭ):

Максимальная кинетическая энергия вылетающих электронов при фотоэффекте может быть выражена через величину задерживающего напряжение Uз и элементарный заряд е:

Существует граничная частота или длинна волны света (называемая красной границей фотоэффекта) такая, что свет с меньшей частотой или большей длиной волны не может вызвать фотоэффект. Эти значения связаны с величиной работы выхода следующим соотношением:

Второй постулат Бора или правило частот (ЗСЭ):

В атоме водорода выполняются следующие соотношения, связывающие радиус траектории вращающегося вокруг ядра электрона, его скорость и энергию на первой орбите с аналогичными характеристиками на остальных орбитах:

На любой орбите в атоме водорода кинетическая (К) и потенциальная (П) энергии электрона связаны с полной энергией (Е) следующими формулами:

Общее число нуклонов в ядре равно сумме числа протонов и нейтронов:

Дефект массы:

Энергия связи ядра выраженная в единицах СИ:

Энергия связи ядра выраженная в МэВ (где масса берется в атомных единицах):

Формула альфа-распада:

Формула бета-распада:

Закон радиоактивного распада:

Ядерные реакции

Для произвольной ядерной реакции описывающейся формулой вида:

Выполняются следующие условия:

Энергетический выход такой ядерной реакции при этом равен:

 

Основы специальной теории относительности (СТО)

К оглавлению…

Релятивистское сокращение длины:

Релятивистское удлинение времени события:

Релятивистский закон сложения скоростей. Если два тела движутся навстречу друг другу, то их скорость сближения:

Релятивистский закон сложения скоростей. Если же тела движутся в одном направлении, то их относительная скорость:

Энергия покоя тела:

Любое изменение энергии тела означает изменение массы тела и наоборот:

Полная энергия тела:

Полная энергия тела Е пропорциональна релятивистской массе и зависит от скорости движущегося тела, в этом смысле важны следующие соотношения:

Релятивистское увеличение массы:

Кинетическая энергия тела, движущегося с релятивистской скоростью:

Между полной энергией тела, энергией покоя и импульсом существует зависимость:

 

Равномерное движение по окружности

К оглавлению…

В качестве дополнения, в таблице ниже приводим всевозможные взаимосвязи между характеристиками тела равномерно вращающегося по окружности (T – период, N – количество оборотов, v – частота, R – радиус окружности, ω – угловая скорость, φ – угол поворота (в радианах), υ – линейная скорость тела, an – центростремительное ускорение, L – длина дуги окружности, t – время):

 

Расширенная PDF версия документа “Все главные формулы по школьной физике”:

К оглавлению…

Формула давления в физике

Содержание:

Определение и формула давления

Определение

Давление – это физическая величина,характеризующая состояние сплошной среды. Оно равно пределу отношения нормальной составляющей силы, которая действует на участок поверхности тела площади $\Delta S$ к размеру данной площади при $\Delta S \rightarrow 0$ . Обозначается давление буквой p. Тогда математической записью определения давления станет формула:

$$p=\lim _{\Delta S \rightarrow 0} \frac{\Delta F_{n}}{\Delta S}=\frac{d F_{n}}{d S}$$

Выражение (1) определяет давление в точке.

Среднее давление

Средним давлением на поверхность называют величину:

$$\langle p\rangle=\frac{F_{n}}{S}(2)$$

где Fn – нормальная составляющая силы, которая действует на рассматриваемую поверхность, S – площадь этой поверхности.

Давление идеального газа

Давление идеального газа вычисляют, используя основное уравнение молекулярно – кинетической теории:

$$p=n k T(3)$$

где $n=\frac{N}{V}$– концентрация молекул газа (N – число частиц), k=1,38•10-23 Дж/К – постоянная Больцмана, T – абсолютная температура газа.{*}$ –поверхностное натяжение жидкости,p0* – давление под не искривлённым слоем жидкости, H – средняя кривизна поверхности жидкости, вычисляемая по закону Лапласа:

$$H=\frac{1}{2}\left(\frac{1}{R_{1}}+\frac{1}{R_{2}}\right)$$

R1, R2 – главные радиусы кривизны.

Единицы измерения давления

Основной единицей измерения давления в системе СИ является: [p]=Па (паскаль)

Внесистемные единицы давления: [p]=мм рт.ст.(миллиметр ртутного столба),мм в.ст (мм водяного столба),атмосфера,бар.

Па= Н/м2 и 1 бар=105 Па.

Техническая атмосфера ~1 бар. Физическая атмосфера 1,01 бар=760 мм рт.ст.. 1 мм рт.ст.=133 Па.

Примеры решения задач

Пример

Задание. Каково давление в море на глубине h=8,5 м, если атмосферное давление равно p0=105 Па, плотность морской воды равна $\rho$=1,03•103 кг/м3

Решение.{5}$ (Па)

Слишком сложно?

Формула давления не по зубам? Тебе ответит эксперт через 10 минут!

Пример

Задание. Каково давление струи на неподвижную плоскость, если струя воды ударяет ее под углом $\alpha$ к нормали плоскости, и упруго отскакивает от нее без изменения скорости? Скорость струи v.

Решение. Сделаем рисунок.

За время $\Delta t$ о стенку ударяется масса воды равная:

$$m=l S \rho=v \Delta t S \rho$$

где S – поперечное сечение струи, $\rho$ – плотность воды. В соответствии с законом сохранения импульса имеем:

$$F \Delta t=m \Delta v \rightarrow F=\frac{m \Delta v}{\Delta t}(2.2)$$

где F – сила, с которой вода действует на стенку.

Примем за положительное направление нормали внешней к опоре и учитывая, что струя отскакивает от стены без потери скорости, получаем:

$$\Delta v=v_{2} \cos \alpha-\left(-v_{1} \cos \alpha\right)=v_{2} \cos \alpha+v_{1} \cos \alpha=2 v \cos \alpha(2.{2}$

Читать дальше: Формула закона Ома.

Сила, Давление – Формулы по физике

По рыхлому снегу человек идёт с большим трудом, глубоко проваливаясь при каждом шаге. Но, надев лыжи, он может идти, почти не проваливаясь в него. Почему? На лыжах или без лыж человек действует на снег с одной и той же силой, равной своему весу. Однако действие этой силы в обоих случаях различно, потому что различна площадь поверхности, на которую давит человек, с лыжами и без лыж. Площадь поверхности лыж почти в 20 раз больше площади подошвы. Поэтому, стоя на лыжах, человек действует на каждый квадратный сантиметр площади поверхности снега с силой, в 20 раз меньшей, чем стоя на снегу без лыж.

Ученик, прикалывая кнопками газету к доске, действует на каждую кнопку с одинаковой силой. Однако кнопка, имеющая более острый конец, легче входит в дерево.


Значит, результат действия силы зависит не только от её модуля, направления и точки приложения, но и от площади той поверхности, к которой она приложена (перпендикулярно которой она действует).

Этот вывод подтверждают физические опыты.

Опыт.Результат действия данной силы зависит от того, какая сила действует на единицу площади поверхности.

По углам небольшой доски надо вбить гвозди. Сначала гвозди, вбитые в доску, установим на песке остриями вверх и положим на доску гирю. В этом случае шляпки гвоздей лишь незначительно вдавливаются в песок. Затем доску перевернем и поставим гвозди на острие. В этом случае площадь опоры меньше, и под действием той же силы гвозди значительно углубляются в песок.

Опыт. Вторая иллюстрация.

От того, какая сила действует на каждую единицу площади поверхности, зависит результат действия этой силы.

В рассмотренных примерах силы действовали перпендикулярно поверхности тела. Вес человека был перпендикулярен поверхности снега; сила, действовавшая на кнопку, перпендикулярна поверхности доски.

Величина, равная отношению силы, действующей перпендикулярно поверхности, к площади этой поверхности, называется давлением.

Чтобы определить давление, надо силу, действующую перпендикулярно поверхности, разделить на площадь поверхности:

давление = сила / площадь.

Обозначим величины, входящие в это выражение: давление – p, сила, действующая на поверхность, – F и площадь поверхности – S.

Тогда получим формулу:

p = F/S

Понятно, что бóльшая по значению сила, действующую на ту же площадь, будет производить большее давление.

За единицу давления принимается такое давление, которое производит сила в 1 Н, действующая на поверхность площадью 1 м2 перпендикулярно этой поверхности.

Единица давления – ньютон на квадратный метр ( 1 Н / м2 ). В честь французского ученого Блеза Паскаля она называется паскалем (Па). Таким образом,

1 Па = 1 Н / м2 .

Используется также другие единицы давления: гектопаскаль (гПа) и килопаскаль (кПа).

1 кПа = 1000 Па;

1 гПа = 100 Па;

1 Па = 0,001 кПа;

1 Па = 0,01 гПа.

Пример. Рассчитать давление, производимое на пол мальчиком, масса которого 45 кг, а площадь подошв его ботинок, соприкасающихся с полом, равна 300 см2.

Запишем условие задачи и решим её.

Дано: m = 45 кг, S = 300 см2; p = ?

В единицах СИ: S = 0,03 м2

Решение:

p = F/S,

F = P,

P = g·m,

P = 9,8 Н · 45 кг ≈ 450 Н,

p = 450/0,03 Н / м2 = 15000 Па = 15 кПа

‘Ответ’: p = 15000 Па = 15 кПа

Способы уменьшения и увеличения давления.

Тяжелый гусеничный трактор производит на почву давление равное 40 – 50 кПа, т. е. всего в 2 – 3 раза больше, чем давление мальчика массой 45 кг. Это объясняется тем, что вес трактора распределяется на бóльшую площадь за счёт гусеничной передачи. А мы установили, что чем больше площадь опоры, тем меньше давление, производимое одной и той же силой на эту опору.

В зависимости от того, нужно ли получить малое или большое давление, площадь опоры увеличивается или уменьшается. Например, для того, чтобы грунт мог выдержать давление возводимого здания, увеличивают площадь нижней части фундамента.

Шины грузовых автомобилей и шасси самолетов делают значительно шире, чем легковых. Особенно широкими делают шины у автомобилей, предназначенных для передвижения в пустынях.

Тяжелые машины, как трактор, танк или болотоход, имея большую опорную площадь гусениц, проходят по болотистой местности, по которой не пройдет человек.

С другой стороны, при малой площади поверхности можно небольшой силой произвести большое давление. Например, вдавливая кнопку в доску, мы действуем на нее с силой около 50 Н. Так как площадь острия кнопки примерно 1 мм2, то давление, производимое ею, равно:

p = 50 Н/ 0, 000 001 м2 = 50 000 000 Па = 50 000 кПа.

Для сравнения, это давление в 1000 раз больше давления, производимого гусеничным трактором на почву. Можно найти еще много таких примеров.

Лезвие режущих и острие колющих инструментов (ножей, ножниц, резцов, пил, игл и др.) специально остро оттачивается. Заточенный край острого лезвия имеет маленькую площадь, поэтому при помощи даже малой силы создается большое давление, и таким инструментом легко работать.

Режущие и колющие приспособления встречаются и в живой природе: это зубы, когти, клювы, шипы и др. – все они из твердого материала, гладкие и очень острые.

Давление

Известно, что молекулы газа беспорядочно движутся.

Опыт. Здесь мы узнаем, что газ давит на стенки сосуда по всем направлениям одинаково.

Мы уже знаем, что газы, в отличие от твердых тел и жидкостей, заполняют весь сосуд, в котором находятся. Например, стальной баллон для хранения газов, камера автомобильной шины или волейбольный мяч. При этом газ оказывает давление на стенки, дно и крышку баллона, камеры или любого другого тела, в котором он находится. Давление газа обусловлено иными причинами, чем давление твердого тела на опору.

Известно, что молекулы газа беспорядочно движутся. При своем движении они сталкиваются друг с другом, а также со стенками сосуда, в котором находится газ. Молекул в газе много, поэтому и число их ударов очень велико. Например, число ударов молекул воздуха, находящегося в комнате, о поверхность площадью 1 см2 за 1 с выражается двадцатитрехзначным числом. Хотя сила удара отдельной молекулы мала, но действие всех молекул на стенки сосуда значительно, — оно и создает давление газа.

Итак, давление газа на стенки сосуда (и на помещенное в газ тело) вызывается ударами молекул газа.

Рассмотрим следующий опыт. Под колокол воздушного насоса поместим резиновый шарик. Он содержит небольшое количество воздуха и имеет неправильную форму. Затем насосом откачиваем воздух из-под колокола. Оболочка шарика, вокруг которой воздух становится все более разреженным, постепенно раздувается и принимает форму правильного шара.

Как объяснить этот опыт?

Для хранения и перевозки сжатого газа используются специальные прочные стальные баллоны.

В нашем опыте движущиеся молекулы газа непрерывно ударяют о стенки шарика внутри и снаружи. При откачивании воздуха число молекул в колоколе вокруг оболочки шарика уменьшается. Но внутри шарика их число не изменяется. Поэтому число ударов молекул о внешние стенки оболочки становится меньше, чем число ударов о внутренние стенки. Шарик раздувается до тех пор, пока сила упругости его резиновой оболочки не станет равной силе давления газа. Оболочка шарика принимает форму шара. Это показывает, что газ давит на ее стенки по всем направлениям одинаково. Иначе говоря, число ударов молекул, приходящихся на каждый квадратный сантиметр площади поверхности, по всем направлениям одинаково. Одинаковое давление по всем направлениям характерно для газа и является следствием беспорядочного движения огромного числа молекул.

Попытаемся уменьшить объем газа, но так, чтобы масса его осталась неизменной. Это значит, что в каждом кубическом сантиметре газа молекул станет больше, плотность газа увеличится. Тогда число ударов молекул о стенки увеличится, т. е. возрастет давление газа. Это можно подтвердить опытом.

На рисунке а изображена стеклянная трубка, один конец которой закрыт тонкой резиновой пленкой. В трубку вставлен поршень. При вдвигании поршня объем воздуха в трубке уменьшается, т. е. газ сжимается. Резиновая пленка при этом выгибается наружу, указывая на то, что давление воздуха в трубке увеличилось.

Наоборот, при увеличении объема этой же массы газа, число молекул в каждом кубическом сантиметре уменьшается. От этого уменьшится число ударов о стенки сосуда – давление газа станет меньше. Действительно, при вытягивании поршня из трубки объем воздуха увеличивается, пленка прогибается внутрь сосуда. Это указывает на уменьшение давления воздуха в трубке. Такие же явления наблюдались бы, если бы вместо воздуха в трубке находился бы любой другой газ.

Итак, при уменьшении объема газа его давление увеличивается, а при увеличении объема давление уменьшается при условии, что масса и температура газа остаются неизменными.

А как изменится давление газа, если нагреть его при постоянном объеме? Известно, что скорость движения молекул газа при нагревании увеличивается. Двигаясь быстрее, молекулы будут ударять о стенки сосуда чаще. Кроме того, каждый удар молекулы о стенку будет сильнее. Вследствие этого, стенки сосуда будут испытывать большее давление.

Следовательно, давление газа в закрытом сосуде тем больше, чем выше температура газа, при условии, что масса газа и объем не изменяются.

Из этих опытов можно сделать общий вывод, что давление газа тем больше, чем чаще и сильнее молекулы ударяют о стенки сосуда.

Для хранения и перевозки газов их сильно сжимают. При этом давление их возрастает, газы необходимо заключать в специальные, очень прочные баллоны. В таких баллонах, например, содержат сжатый воздух в подводных лодках, кислород, используемый при сварке металлов. Конечно же, мы должны навсегда запомнить, что газовые баллоны нельзя нагревать, тем более, когда они заполнены газом. Потому что, как мы уже понимаем, может произойти взрыв с очень неприятными последствиями.

Закон Паскаля.

Давление передается в каждую точку жидкости или газа.

Давление поршня передается в каждую точку жидкости, заполняющей шар.

Теперь газ.

В отличие от твердых тел отдельные слои и мелкие частицы жидкости и газа могут свободно перемещаться относительно друг друга по всем направлениям. Достаточно, например, слегка подуть на поверхность воды в стакане, чтобы вызвать движение воды. На реке или озере при малейшем ветерке появляется рябь.

Подвижностью частиц газа и жидкости объясняется, что давление, производимое на них, передается не только в направлении действия силы, а в каждую точку. Рассмотрим это явление подробнее.

На рисунке, а изображен сосуд, в котором содержится газ (или жидкость). Частицы равномерно распределены по всему сосуду. Сосуд закрыт поршнем, который может перемещаться вверх и вниз.

Прилагая некоторую силу, заставим поршень немного переместиться внутрь и сжать газ (жидкость), находящийся непосредственно под ним. Тогда частицы (молекулы) расположатся в этом месте более плотно, чем прежде(рис, б). Благодаря подвижности частицы газа будут перемещаться по всем направлениям. Вследствие этого их расположение опять станет равномерным, но более плотным, чем раньше (рис, в). Поэтому давление газа всюду возрастет. Значит, добавочное давление передается всем частицам газа или жидкости. Так, если давление на газ (жидкость) около самого поршня увеличится на 1 Па, то во всех точках внутри газа или жидкости давление станет больше прежнего на столько же. На 1 Па увеличится давление и на стенки сосуда, и на дно, и на поршень.

Давление, производимое на жидкость или газ, передается на любую точку одинаково во всех направлениях.

Это утверждение называется законом Паскаля.

На основе закона Паскаля легко объяснить следующие опыты.

На рисунке изображен полый шар, имеющий в различных местах небольшие отверстия. К шару присоединена трубка, в которую вставлен поршень. Если набрать воды в шар и вдвинуть в трубку поршень, то вода польется из всех отверстий шара. В этом опыте поршень давит на поверхность воды в трубке. Частицы воды, находящиеся под поршнем, уплотняясь, передают его давление другим слоям, лежащим глубже. Таким образом, давление поршня передается в каждую точку жидкости, заполняющей шар. В результате часть воды выталкивается из шара в виде одинаковых струек, вытекающих из всех отверстий.

Если шар заполнить дымом, то при вдвигании поршня в трубку из всех отверстий шара начнут выходить одинаковые струйки дыма. Это подтверждает, что и газы передают производимое на них давление во все стороны одинаково.

Давление в жидкости и газе.

Под действием веса жидкости резиновое дно в трубке прогнется.

На жидкости, как и на все тела на Земле, действует сила тяжести. Поэтому, каждый слой жидкости, налитой в сосуд, своим весом создает давление, которое по закону Паскаля передается по всем направлениям. Следовательно, внутри жидкости существует давление. В этом можно убедиться на опыте.

В стеклянную трубку, нижнее отверстие которой закрыто тонкой резиновой пленкой, нальем воду. Под действием веса жидкости дно трубки прогнется.

Опыт показывает, что, чем выше столб воды над резиновой пленкой, тем больше она прогибается. Но всякий раз после того, как резиновое дно прогнулось, вода в трубке приходит в равновесие (останавливается), так как, кроме силы тяжести, на воду действует сила упругости растянутой резиновой пленки.

По мере опускания трубки

резиновая пленка постепенно выпрямляется.

Силы, действующие на резиновую пленку,

одинаковы с обеих сторон.

Иллюстрация.

Дно отходит от цилиндра вследствие давления на него силы тяжести.

Опустим трубку с резиновым дном, в которую налита вода, в другой, более широкий сосуд с водой. Мы увидим, что по мере опускания трубки резиновая пленка постепенно выпрямляется. Полное выпрямление пленки показывает, что силы, действующие на нее сверху и снизу, равны. Наступает полное выпрямление пленки тогда, когда уровни воды в трубке и сосуде совпадают.

Такой же опыт можно провести с трубкой, в которой резиновая пленка закрывает боковое отверстие, как это показано на рисунке, а. Погрузим эту трубку с водой в другой сосуд с водой, как это изображено на рисунке, б. Мы заметим, что пленка снова выпрямится, как только уровни воды в трубке и сосуде сравняются. Это означает, что силы, действующие на резиновую пленку, одинаковы со всех сторон.

Возьмем сосуд, дно которого может отпадать. Опустим его в банку с водой. Дно при этом окажется плотно прижатым к краю сосуда и не отпадет. Его прижимает сила давления воды, направленная снизу вверх.

Будем осторожно наливать воду в сосуд и следить за его дном. Как только уровень воды в сосуде совпадет с уровнем воды в банке, оно отпадет от сосуда.

В момент отрыва на дно давит сверху вниз столб жидкости в сосуде, а снизу вверх на дно передается давление такого же по высоте столба жидкости, но находящейся в банке. Оба эти давления одинаковы, дно же отходит от цилиндра вследствие действия на него собственной силы тяжести.

Выше были описаны опыты с водой, но если взять вместо воды любую другую жидкость, результаты опыта будут те же.

Итак, опыты показывают, что внутри жидкости существует давление, и на одном и том же уровне оно одинаково по всем направлениям. С глубиной давление увеличивается.

Газы в этом отношении не отличаются от жидкостей, ведь они тоже имеют вес. Но надо помнить, что плотность газа в сотни раз меньше плотности жидкости. Вес газа, находящегося в сосуде, мал, и его “весовое” давление во многих случаях можно не учитывать.

Формулы по физике для ЕГЭ и 7-11 класса

Рубрика: Подготовка к ЕГЭ по физике

Шпаргалка с формулами по физике для ЕГЭ

и не только (может понадобиться 7, 8, 9, 10 и 11 классам).

Для начала картинка, которую можно распечатать в компактном виде.

Механика

  1. Давление                      Р=F/S
  2. Плотность                   ρ=m/V
  3. Давление на глубине жидкости   P=ρ∙g∙h
  4. Сила тяжести                       Fт=mg
  5. 5. Архимедова сила                 Fa=ρж∙g∙Vт
  6. Уравнение движения  при равноускоренном  движении

X=X0+υ0∙t+(a∙t2)/2                    S= (υ2υ02)/2а         S= (υ+υ0) ∙t /2

  1. Уравнение скорости  при равноускоренном движении υ=υ0+a∙t
  2. Ускорение            a=(υυ 0)/t
  3. Скорость при движении по окружности υ=2πR/Т
  4. Центростремительное ускорение  a=υ2/R
  5. Связь периода с частотой ν=1/T=ω/2π
  6. II закон Ньютона                F=ma
  7. Закон Гука                          Fy=-kx
  8. Закон Всемирного тяготения  F=G∙M∙m/R2
  9. Вес тела, движущегося с ускорением а↑      Р=m(g+a)
  10. Вес тела, движущегося с ускорением а↓      Р=m(g-a)
  11. Сила трения                     Fтр=µN
  12. Импульс тела                       p=mυ
  13. Импульс силы                     Ft=∆p
  14. Момент силы                    M=F∙ℓ
  15. Потенциальная энергия тела, поднятого над землей Eп=mgh
  16. Потенциальная энергия упруго деформированного тела Eп=kx2/2
  17. Кинетическая энергия тела Ek=mυ2/2
  18. Работа            A=F∙S∙cosα
  19. Мощность     N=A/t=F∙υ
  20. Коэффициент полезного действия η=Aп/Аз
  21. Период колебаний математического маятника T=2π√ℓ/g
  22. Период колебаний пружинного маятника T=2 π √m/k
  23. Уравнение гармонических колебаний  Х=Хmax∙cos ωt
  24. Связь длины волны, ее скорости и периода λ= υТ

Молекулярная физика и термодинамика

  1. Количество вещества              ν=N/ Na
  2. Молярная масса                           М=m/ν
  3. Cр. кин. энергия молекул одноатомного газа Ek=3/2∙kT
  4. Основное уравнение МКТ      P=nkT=1/3nm0υ2
  5. Закон Гей – Люссака (изобарный процесс)    V/T =const
  6. Закон Шарля (изохорный процесс)    P/T =const
  7. Относительная влажность φ=P/P0∙100%
  8. Внутр. энергия идеал. одноатомного газа U=3/2∙M/µ∙RT
  9. Работа газа A=P∙ΔV
  10. Закон Бойля – Мариотта (изотермический процесс)    PV=const
  11. Количество теплоты при нагревании  Q=Cm(T2-T1)
  12. Количество теплоты при плавлении   Q=λm
  13. Количество теплоты при парообразовании  Q=Lm
  14. Количество теплоты при сгорании топлива  Q=qm
  15. Уравнение состояния идеального газа PV=m/M∙RT
  16. Первый закон термодинамики   ΔU=A+Q
  17. КПД тепловых двигателей         η= (Q1 – Q2)/ Q1
  18. КПД идеал. двигателей  (цикл Карно)     η= (Т1 – Т2)/ Т1

https://5-ege.ru/formuly-po-fizike-dlya-ege/

Электростатика и электродинамика – формулы по физике

  1. Закон Кулона F=k∙q1∙q2/R2
  2. Напряженность электрического поля E=F/q
  3. Напряженность эл. поля точечного заряда E=k∙q/R2
  4. Поверхностная плотность зарядов             σ = q/S
  5. Напряженность эл. поля бесконечной плоскости E=2πkσ
  6. Диэлектрическая проницаемость ε=E0/E
  7. Потенциальная энергия взаимод. зарядов W= k∙q1q2/R
  8. Потенциал φ=W/q
  9. Потенциал точечного заряда φ=k∙q/R
  10. Напряжение U=A/q
  11. Для однородного электрического поля U=E∙d
  12. Электроемкость C=q/U
  13. Электроемкость плоского конденсатора C=S∙εε0/d
  14. Энергия заряженного конденсатора W=qU/2=q²/2С=CU²/2
  15. Сила тока I=q/t
  16. Сопротивление проводника R=ρ∙ℓ/S
  17. Закон Ома для участка цепи I=U/R
  18. Законы послед. соединения I1=I2=I, U1+U2=U, R1+R2=R
  19. Законы паралл. соед.   U1=U2=U, I1+I2=I, 1/R1+1/R2=1/R
  20. Мощность электрического тока P=I∙U
  21. Закон Джоуля-Ленца Q=I2Rt
  22. Закон Ома для полной цепи I=ε/(R+r)
  23. Ток короткого замыкания (R=0)      I=ε/r
  24. Вектор магнитной индукции B=Fmax/ℓ∙I
  25. Сила Ампера Fa=IBℓsin α
  26. Сила Лоренца Fл=Bqυsin α
  27. Магнитный поток Ф=BSсos α      Ф=LI
  28. Закон электромагнитной индукции Ei=ΔФ/Δt
  29. ЭДС индукции в движ проводнике Ei=Вℓυsinα
  30. ЭДС самоиндукции Esi=-L∙ΔI/Δt
  31. Энергия магнитного поля катушки Wм=LI2/2
  32. Период колебаний кол. контура T=2π ∙√LC
  33. Индуктивное сопротивление XL=ωL=2πLν
  34. Емкостное сопротивление Xc=1/ωC
  35. Действующее значение силы тока Iд=Imax/√2,
  36. Действующее значение напряжения Uд=Umax/√2
  37. Полное сопротивление Z=√(Xc-XL)2+R2

Оптика

  1. Закон преломления света     n21=n2/n1= υ 1/ υ 2
  2. Показатель преломления      n21=sin α/sin γ
  3. Формула тонкой линзы       1/F=1/d + 1/f
  4. Оптическая сила линзы       D=1/F
  5. max интерференции: Δd=kλ,
  6. min интерференции: Δd=(2k+1)λ/2
  7. Диф.решетка             d∙sin φ=k λ

Квантовая физика

  1. Ф-ла Эйнштейна для фотоэффекта  hν=Aвых+Ek, Ek=Uзе
  2. Красная граница фотоэффекта νк = Aвых/h
  3. Импульс фотона P=mc=h/ λ=Е/с

Физика атомного ядра

  1. Закон радиоактивного распада N=N0∙2t/T
  2. Энергия связи атомных ядер

ECB=(Zmp+Nmn-Mя)∙c2

СТО

  1. t=t1/√1-υ2/c2
  2. ℓ=ℓ0∙√1-υ2/c2
  3. υ2=(υ1+υ)/1+ υ1∙υ/c2
  4. Е = mс2

Скачать эти формулы в doc: formuly-po-fizike-5-ege.ru (файл расположен на 5-ege.ru).

Рекомендуем:

Что такое давление, давление в физике, формула давления

      Здравствуйте! Сегодня мы поговорим о такой физической величине и описываем ею явлении как давление. Я расскажу о том, откуда оно возникает, и какие внешние проявления давления вы можете наблюдать в жизни и в быту.

Давление как физическая величина

     Физическая величина давление равна силе приходящейся на единицу площади. Вообще отношение силы, давления и площади можно выразить в виде вот такого мнемонического треугольника как на рисунке ниже.

сила-давление-площадь

Как мы видим из него

p=F/S,

S=F/p,

F=p*S.

     Давление в физике измеряется в Паскалях (Па) и имеет размерность Н/кв. м. Как видно и из формулы и из определения самого давления, данная величина характеризует меру воздействия сил, приходящуюся на единицу площади.

И как видно из формулы, чтобы увеличить величину давления, нужно либо увеличить прилагаемую силу, либо уменьшить площадь, на которую приложена сила. Запомните пока это обстоятельство, т.к. оно нам пригодится для объяснения различных природных и бытовых явлений связанных с давлением.

Давление как явление природы

     У меня для вас сразу вопрос на засыпку: может ли человек произвести с помощью только своих рук давление больше 1000 атмосфер? Как на странно, но ответ на этот вопрос: да может! А теперь докажем это утверждение расчетами.

    Итак, одна атмосфера равна давлению в 100000 Па. Сила давления пальца человека на острое шило где-то 30 Н (можно и больше). Площадь острия шила составляет около 0,03 квадратных миллиметра. Т.е. это 0,00000003 кв. м. Разделим 30 Н на 0,00000003 кв. м. и получим 1000000000 Па. Или 10000 атм. Чудовищное давление, не правда ли?

Для сравнения я приведу некоторые данные:

Давление в автомобильных шинах – 2 атм.

Давление в камере бензинового двигателя – 10 атм.

Давление в камере дизеля – 20 атм.

Давление в паровом котле – 40 атм.

     Правда тут есть одно НО. В приведенных примерах давление внутри этих устройств распределено по ВСЕ рабочей площади этих устройств, а вот в приведенном нами примере все это чудовищное давление сосредоточено на кончике шила (0,03 кв. мм).

     Данные вычисления объясняют, почему острыми иголками и ножами можно проткнуть или разрезать довольно прочные предметы. Тогда как с тупыми иголками и ножами это сделать проблематично.

     Но это мы все говорили о высоких давлениях. А вот иногда нужно наоборот давление уменьшить. Вы наверно сами уже догадались, как это сделать. Конечно же, увеличить площадь соприкосновения сил. Как пример такого увеличения площади, для уменьшения давления можно привести лыжи, болотные «мокроступы», гусеницы трактора и даже обыкновенный мягкий матрас.

     Вы скажете, а причем тут матрас? А вы пробовали когда-нибудь спать на ровном и жестком полу? Так вот обычный матрас увеличивает площадь соприкосновения тела с поверхностью и поэтому лежать на нем мягко. А вот на ровном полу мы касаемся его лишь небольшими участками тела, и как следствие давление на эти части приходится значительно большее. Вот именно поэтому на ровной поверхности можно себе чего-нибудь «отлежать».


просто и понятно о том, как определяется давление

Определение давления в физике
  • Общая формула давления

  • Единицы давления

  • Формула гидростатического давления

  • Парциальное давление и его формула

  • Формула давления идеального газа

  • Приборы для измерения давления

  • Рекомендованная литература и полезные ссылки

  • Расчет давления жидкости на дно и стенки сосуда, видео
  • Давление – очень важная физическая величина, играющая огромную роль, как в окружающей природе, так и жизни человека. Внешне незаметное человеческому глазу давление может очень хорошо ощущаться каждым из нас. Особенно хорошо это усвоили люди в возрасте, часто страдающие от повышенного давления (или наоборот от пониженного). Но в нашей статье мы больше поговорим именно о давлении в физике, о том, как оно измеряется и рассчитывается, какие есть формулы для расчетов давления разных субстанций: воздуха, жидкости или твердого тела.

    Определение давления в физике

    Под давлением в физике понимается термодинамическая величина, выраженная соотношением перпендикулярной силы давления на площадь поверхности, на которую она воздействует. При этом согласно закону Паскаля если система находится в состоянии равновесия, то давление на нее будет одинаковым для всех точек системы.

    В физике, как впрочем и химии, давление обозначают большой буквой Р, идущей от латинского слова «pressura» – давление. (В английском языке давление так и осталось почти без изменения – pressure).

    Общая формула давления

    Из классического определения того, что такое давление можно вывести общую формулу для его расчета. Выглядеть она будет таким образом:

    P = F/S

    Где F – это сила давления, а S – площадь поверхности на которую она действует. То есть иными словами формула нахождения давления – это сила, воздействующая на определенную поверхность, разделенная на площадь этой самой поверхности.

    Как видно из формулы, при расчете давления всегда действует следующий принцип: чем меньше пространство, на которое влияет сила, тем большее количество давящей силы на него приходится и наоборот.

    Это можно проиллюстрировать простым жизненным примером: хлеб легче всего порезать острым ножом, потому что у острого ножа заточенное лезвие, то есть площадь поверхности S из формулы у него минимальна, а значит, давление ножа на хлеб будет максимально равно приложенной силе F того кто держит нож. А вот тупым ножом порезать хлеб уже сложнее, так как у его лезвия большая площадь поверхности S, и давление ножа на хлеб будет меньшим, и значит, чтобы отрезать себе кусок хлеба нужно приложить большее количество силы F.

    Общая формула давления, по сути, отлично описывает формулу давления твердого тела.

    Единицы давления

    Согласно стандартам Международной метрической системы давление измеряется в паскалях. Один паскаль из классической формулы равен одному Ньютону (Как мы знаем, Ньютон у нас единица измерения силы) разделенному на один квадратный метр.

    Но увы на практике паскаль оказывается очень маленькой единицей и использовать его для измерения давления не всегда удобно, поэтому часто для измерения давления применяют другие единицы:

    • Бары – один бар равен 105 паскалей
    • Миллиметры водяного столпа
    • Метры водяного столпа
    • Технические и физические атмосферы

    Формула гидростатического давления

    Как мы знаем, разные агрегатные состояния вещества, имеют разные физические свойства. Жидкости своими свойствами отличаются от твердых тел, а газы в свою очередь отличаются от них всех. Поэтому вполне логично, что способы определения давления для жидкостей, твердых тел и газов также будут разными. Так, например, формула давления воды (или гидростатического давления) будет иметь следующий вид:

    P = p*g*h

    Где маленькая p – плотность вещества, g – ускорение свободного падения, h – высота.

    В частности эта формула объясняет, почему при погружении водолазов (или батискафа или подводной лодки) на глубину все больше возрастает давление окружающей воды. Также из этой формулы понятно, почему на предмет, погруженный в какой-нибудь кисель, будет воздействовать большее давление, чем на предмет, погруженный просто в воду, так как плотность киселя (p) выше, чем у воды, а чем выше плотность жидкости, тем выше ее гидростатическое давление.

    Приведенная нами формула гидростатического давления справедлива не только для жидкостей, но и для газов. Поэтому поднимаясь высоко в горы (где воздух более разрежен, а значит меньшее давление), как и спускаясь в подводные глубины, человек, водолаз или альпинист должен пройти специальную адаптацию, привыкнуть к тому, что на него будет воздействовать другое давление.

    Резкая смена давления может привести к кессоной болезни (в случае с водолазами) или к «горной» болезни (в случае с альпинистами). И «кесонка» и «горняшка», как их сленгово называют водолазы и альпинисты, вызвана резкой сменной давления окружающей среды. То есть, если не подготовленный человек начнет вдруг подниматься на Эверест, то он быстро словит «горняшку», а если этот же человек начнет опускаться на дно Мариинской впадины, то гарантировано получит «кесонку». В первом случае причиной будет не адаптация организма к пониженному давлению, а во втором – к повышенному.

    Американские водолазы в декомпрессионой камере, призванной подготовить их к глубоководным погружениям и адаптировать организм к высокому давлению океанских глубин.

    Парциальное давление и его формула

    Хотя формула гидростатического давления применима для газов, но давления для них удобнее вычислять по другой формуле, формуле парциального давления.

    Дело в том, что в природе редко встречаются абсолютно чистые вещества, причем это касается как жидкостей, так и газов. Обычно на практике в окружающем мире преобладают различные смеси, и логично, что каждый из компонентов такой смеси может оказывать разное давление, такое разное давление и называют парциальным. Определить парциальное давление просто – оно равно суме давлений каждого компонента рассматриваемой смеси. Отсюда формула парциального давления будет иметь следующий вид:

    P = P1+P2+P3

    Где P1, P2 и P3 – давления каждого из компонентов газовой смеси, так званный «идеальный газ».

    К примеру, чтобы определить давления воздуха обычной формулы гидростатического давления проделанной только с кислородом недостаточно, так как воздух в реальности представляет собой смесь разных газов, где помимо основного компонента кислорода, которым мы все дышим, есть и другие: азот, аргон и т. д.

    Такие расчеты нужно проделывать при помощи формулы парциального давления.

    Формула давления идеального газа

    Также стоит заметить, что давление идеального газа, то есть каждого отдельного из компонентов газовой смеси удобно посчитать по формуле молекулярно-кинетической теории.

    P = n*k*T

    Где n – концентрация молекул газа, T – абсолютная температура газа, k – постоянная Больцмана (указывает на взаимосвязь между кинетической энергией частицы газа и ее абсолютной температурой), она равна 1,38*10-23 Дж/К.

    Приборы для измерения давления

    Разумеется, человечество изобрело многие приборы, позволяющие быстро и удобно измерять уровень давления. Для измерения давления окружающей среды, оно же атмосферное давление используют такой прибор как манометр или барометр.

    Так выглядит классический барометр для измерения атмосферного давления.

    Чтобы узнать артериальное давление у человека, часто служащее причиной недомоганий используется прибор известный большинству под названием неинвазивный тонометр. Таких приборов существует множество разновидностей.

    Также биологи в своих исследованиях занимаются расчетами осмотического давления – это давление внутри и снаружи клетки. А метеорологи, в частности по перепадам давления в окружающей среде предсказывают нам погоду.

    Рекомендованная литература и полезные ссылки

    • Кузнецов В. Н. Давление. Большая Российская Энциклопедия. Дата обращения 27 августа 2016.
    • E.R. Cohen et al, «Quantities, Units and Symbols in Physical Chemistry», IUPAC Green Book, 3rd Edition, 2nd Printing, IUPAC & RSC Publishing, Cambridge (2008). — p. 14.

    Расчет давления жидкости на дно и стенки сосуда, видео


    Автор: Павел Чайка, главный редактор журнала Познавайка

    При написании статьи старался сделать ее максимально интересной, полезной и качественной. Буду благодарен за любую обратную связь и конструктивную критику в виде комментариев к статье. Также Ваше пожелание/вопрос/предложение можете написать на мою почту [email protected] или в Фейсбук, с уважением автор.


    Формула давления

    ОПРЕДЕЛЕНИЕ

    Давление столба жидкости (гидростатическое давление) равно плотности этой жидкости, умноженной на высоту столба жидкости и ускорение свободного падения.

       

    Здесь – давление, – плотность жидкости, – ускорение свободного падения ( м/с), – высота столба жидкости (глубина, на которой находится сдавливаемое тело).

    Единица измерения давления – Па (паскаль).

    Это векторная величина. В каждой точке жидкости давление одинаково во всех направлениях. Чаще всего в задачах требуется найти давление столба воды. Её плотность – 1000 кг/м. Формула верна не только для жидкости, но и для идеального газа. Есть ещё одна формула давления:

       

    Где – сила тяжести, действующая на жидкость (её вес), – площадь поверхности, на которую оказывается давление.

    Примеры решения задач по теме «Давление»

    ПРИМЕР 1
    Задание Высота воды в аквариуме 1 м. Найти давление на дно аквариума.
    Решение Напоминаем, плотность воды кг/м, а м/с. Таким образом:

    (Па)

    Ответ Давление воды составляет 9800 Паскаль.
    ПРИМЕР 2
    Задание Закрытый сосуд устроен так, что его крышка подвижна (см. рисунок). Сосуд наполнен жидкостью, плотность которой известна. На крышку сосуда действует сила . Найти давление на дно сосуда, если известны площадь дна и крышки и объём жидкости в сосуде.
    Решение Пусть:

    – площадь крышки

    – площадь дна

    – объём жидкости

    – плотность жидкости

    Крышка подвижна, значит она давит на жидкость с той силой, с которой внешние силы давят на неё сверху.

    Очевидно:

    Попробуем найти давление жидкости:

    Соберём всё вместе:

    Ответ
    Читайте также:

    Все формулы по физике

    Формула силы выталкивания

    Формула напряжённости магнитного поля

    Формула силы Ампера

    Формула силы Лоренца

    Формула ЭДС

    Формула длины волны

    Линейный импульс и сила | Физика

    Цели обучения

    К концу этого раздела вы сможете:

    • Определите количество движения.
    • Объясните взаимосвязь между импульсом и силой.
    • Укажите второй закон движения Ньютона с точки зрения количества движения.
    • Вычислить импульс с учетом массы и скорости.

    Линейный импульс

    Научное определение количества движения согласуется с интуитивным пониманием количества движения большинства людей: большой, быстро движущийся объект имеет больший импульс, чем меньший и более медленный объект. Линейный импульс определяется как произведение массы системы на ее скорость. В символах линейный импульс выражается как p = м v .

    Импульс прямо пропорционален массе объекта, а также его скорости. Таким образом, чем больше масса объекта или чем больше его скорость, тем больше его импульс. Импульс p – это вектор, имеющий то же направление, что и скорость v . Единица измерения количества движения в системе СИ – кг · м / с.

    Линейный импульс

    Линейный импульс определяется как произведение массы системы на ее скорость:

    p = м v

    Пример 1. Расчет импульса: футболист и футбол

    1. Рассчитайте импульс футболиста весом 110 кг, бегущего со скоростью 8,00 м / с.
    2. Сравните импульс игрока с импульсом сильно брошенного футбольного мяча весом 0,410 кг, который имеет скорость 25,0 м / с.
    Стратегия

    Никакой информации относительно направления не дается, поэтому мы можем вычислить только величину импульса, p .(Как обычно, курсивом обозначена величина, а курсивом, полужирным шрифтом и стрелкой – вектор.) В обеих частях этого примера величина импульса может быть вычислена непосредственно из определения числа импульс, указанный в уравнении, который становится p = mv , если рассматривать только величины.

    Решение для части 1

    Чтобы определить импульс игрока, подставьте известные значения массы и скорости игрока в уравнение.

    p игрок = (110 кг) (8,00 м / с) = 880 кг · м / с

    Решение для части 2

    Чтобы определить импульс мяча, подставьте известные значения массы и скорости мяча в уравнение.

    p мяч = (0,410 кг) (25,0 м / с) = 10,3 кг · м / с

    Отношение количества движения игрока к импульсу мяча равно

    .

    [латекс] \ displaystyle \ frac {p _ {\ text {player}}} {p _ {\ text {ball}}} = \ frac {880} {10.3} = 85,9 \ [/ латекс]

    Обсуждение

    Хотя мяч имеет большую скорость, игрок имеет гораздо большую массу. Таким образом, как вы могли догадаться, импульс игрока намного больше, чем импульс футбола. В результате, если игрок ловит мяч, это лишь незначительно влияет на его движение. В следующих разделах мы дадим количественную оценку тому, что происходит при таких столкновениях, с точки зрения количества движения.

    Импульс и второй закон Ньютона

    Важность импульса, в отличие от энергии, была признана на раннем этапе развития классической физики.Импульс считался настолько важным, что его называли «количеством движения». Ньютон фактически сформулировал свой второй закон движения в терминах количества движения: чистая внешняя сила равна изменению количества движения системы, деленному на время, в течение которого он изменяется. Используя символы, этот закон равен

    .

    [латекс] \ displaystyle {\ mathbf {F}} _ {\ text {net}} = \ frac {\ Delta \ mathbf {p}} {\ Delta t} [/ latex],

    , где F net – чистая внешняя сила, Δp – изменение количества движения, а Δ t – изменение во времени.

    Второй закон движения Ньютона по импульсу

    Чистая внешняя сила равна изменению количества движения системы, деленному на время, в течение которого он изменяется.

    [латекс] \ displaystyle {\ mathbf {F}} _ {\ text {net}} = \ frac {\ Delta \ mathbf {p}} {\ Delta t} [/ latex]

    Установление связей: сила и импульс

    Сила и импульс тесно связаны. Сила, действующая с течением времени, может изменять импульс, и второй закон движения Ньютона может быть сформулирован в его наиболее широко применимой форме с точки зрения количества движения.Импульс продолжает оставаться ключевым понятием при изучении атомных и субатомных частиц в квантовой механике.

    Это утверждение второго закона движения Ньютона включает более знакомую F net = m a как частный случай. Мы можем получить эту форму следующим образом. Во-первых, обратите внимание, что изменение импульса Δp определяется как Δp = Δ ( м v).

    Если масса системы постоянна, то Δ ( м v) = м Δv.

    Таким образом, для постоянной массы второй закон движения Ньютона принимает вид

    [латекс] \ displaystyle {\ mathbf {F}} _ {\ text {net}} = \ frac {\ Delta \ mathbf {p}} {\ Delta t} = \ frac {m \ Delta \ mathbf {v} } {\ Delta {t}} [/ latex]

    Поскольку [latex] \ frac {\ Delta \ mathbf {v}} {\ Delta {t}} = \ mathbf {a} \\ [/ latex], мы получаем знакомое уравнение F net = m a при постоянной массе системы .

    Второй закон движения Ньютона, выраженный в единицах количества движения, более широко применим, поскольку его можно применять к системам с изменяющейся массой, таким как ракеты, а также к системам с постоянной массой. Рассмотрим подробнее системы с разной массой; однако связь между импульсом и силой остается полезной, когда масса постоянна, как в следующем примере.

    Пример 2. Расчетное усилие: ракетка Винус Уильямс

    Во время Открытого чемпионата Франции 2007 года Винус Уильямс показала самую быструю подачу в женском матче, достигнув скорости 58 м / с (209 км / ч).Какова средняя сила, прилагаемая ракеткой Винус Уильямс к теннисному мячу весом 0,057 кг, если предположить, что скорость мяча сразу после удара составляет 58 м / с, что начальная горизонтальная составляющая скорости до удара пренебрежимо мала и что мяч оставался в контакте с ракеткой в ​​течение 5,0 мс (миллисекунд)?

    Стратегия

    Эта проблема включает только одно измерение, потому что мяч не имеет горизонтальной составляющей скорости до удара. Второй закон Ньютона, выраженный в единицах количества движения, записывается как

    [латекс] \ displaystyle {\ mathbf {F}} _ {\ text {net}} = \ frac {\ Delta \ mathbf {p}} {\ Delta t} [/ latex]

    Как отмечалось выше, когда масса постоянна, изменение количества движения определяется выражением Δ p = м Δ v = м ( v f v i ).

    В этом примере даны скорость сразу после удара и изменение во времени; таким образом, после вычисления Δ p можно использовать [latex] {\ mathbf {F}} _ {\ text {net}} = \ frac {\ Delta {p}} {\ Delta t} [/ latex] найти силу.

    Решение

    Чтобы определить изменение количества движения, подставьте значения начальной и конечной скоростей в приведенное выше уравнение.

    [латекс] \ begin {array} {lll} \ Delta {p} & = & m (v _ {\ text {f}} – v {\ text {i}}) \\ & = & (0,057 \ text {кг }) (58 \ текст {м / с} -0 \ текст {м / с}) \\ & = & 3.{-3} \ text {s}} \\ & = & 661 \ text {N} \ приблизительно660 \ text {N} \ end {array} \\ [/ latex]

    , где на последнем этапе мы оставили только две значащие цифры.

    Обсуждение

    Эта величина представляла собой среднюю силу, прилагаемую ракеткой Винус Уильямс к теннисному мячу во время его кратковременного удара (обратите внимание, что мяч также испытал силу тяжести 0,56 Н, но эта сила возникла не из-за ракетки). Эту проблему также можно решить, сначала найдя ускорение, а затем используя F net = ma , но потребуется один дополнительный шаг по сравнению со стратегией, использованной в этом примере.

    Сводка раздела

    • Линейный импульс (для краткости) определяется как произведение массы системы на ее скорость.
    • В символах, импульс p определен как p = m v , где m – масса системы, а v – ее скорость.
    • Единица измерения количества движения в системе СИ – кг · м / с.
    • Второй закон движения Ньютона с точки зрения количества движения гласит, что чистая внешняя сила равна изменению количества движения системы, деленному на время, в течение которого он изменяется.
    • В символах второй закон движения Ньютона определяется как [latex] {\ mathbf {F}} _ {\ text {net}} = \ frac {\ Delta \ mathbf {p}} {\ Delta t} \\ [/ latex], F net – чистая внешняя сила, Δ p – изменение количества движения, а Δ t – время изменения.

    Концептуальные вопросы

    1. Объект с малой массой и объект с большой массой имеют одинаковый импульс. Какой объект имеет наибольшую кинетическую энергию?
    2. Объект с малой массой и объект с большой массой имеют одинаковую кинетическую энергию.Какая масса имеет наибольший импульс?
    3. Профессиональное приложение. Футбольные тренеры советуют игрокам блокировать, бить и отбиваться ногами на земле, а не прыгать в воздухе. Используя концепции импульса, работы и энергии, объясните, как футболист может быть более эффективным, стоя на земле.
    4. Как малая сила может передать объекту такой же импульс, что и большая сила?

    Задачи и упражнения

    1. (a) Вычислите импульс 2000-кг слона, который атакует охотника со скоростью 7.50 м / с. (б) Сравните импульс слона с импульсом дротика с транквилизатором весом 0,0400 кг, выпущенного со скоростью 600 м / с. (c) Каков импульс 90-килограммового охотника, бегущего со скоростью 7,40 м / с после того, как пропустил слона?
    2. (а) Какова масса большого корабля с импульсом 1,60 × 10 9 кг · м / с, когда корабль движется со скоростью 48,0 км / ч? (b) Сравните импульс корабля с импульсом артиллерийского снаряда массой 1100 кг, выпущенного со скоростью 1200 м / с.
    3. (a) С какой скоростью будет 2.00 × 10 4 -кг самолет должен лететь, чтобы иметь импульс 1,60 × 10 9 кг · м / с (такой же, как импульс корабля в задаче выше)? (b) Какова инерция самолета, когда он взлетает со скоростью 60,0 м / с? (c) Если корабль является авианосцем, который запускает эти самолеты с помощью катапульты, обсудите последствия вашего ответа на (b), поскольку он относится к эффектам отдачи катапульты на корабль.
    4. (a) Каков импульс мусоровоза, который составляет 1,20 × 10 4 кг и движется со скоростью 10.0 м / с? (b) На какой скорости мусор весом 8 кг может иметь такую ​​же скорость, что и грузовик?
    5. Неуправляемый вагон массой 15 000 кг движется по рельсам со скоростью 5,4 м / с. Вычислите время, необходимое для того, чтобы заставить автомобиль остановиться, приложив усилие в 1500 Н.
    6. Масса Земли составляет 5,972 × 10 24 кг, а ее орбитальный радиус составляет в среднем 1,496 × 10 11 м. Рассчитайте его импульс.

    Глоссарий

    Количество движения: произведение массы на скорость

    второй закон движения: физический закон, который гласит, что чистая внешняя сила равна изменению количества движения системы, деленному на время, в течение которого оно изменяется

    Избранные решения проблем и упражнения

    1.а) 1,50 × 10 4 кг м / с; (б) 625 к 1; (в) 6,66 × 10 2 кг ⋅ м / с

    3. (а) 8.00 × 10 4 м / с; (б) 1,20 × 10 6 кг · м / с; (c) Поскольку импульс самолета на 3 порядка меньше, чем у корабля, корабль не будет сильно отскакивать. Отдача составит -0,0100 м / с, что, вероятно, не заметно.

    5. 54 с

    Мощность

    Количественная работа связана с силой, вызывающей смещение.Работа не имеет ничего общего с количеством времени, в течение которого эта сила вызывает смещение. Иногда работа выполняется очень быстро, а иногда – довольно медленно. Например, альпинистке требуется необычно много времени, чтобы поднять свое тело на несколько метров вдоль скалы. С другой стороны, турист (который выберет более легкий путь в гору) может поднять свое тело на несколько метров за короткий промежуток времени. Эти два человека могут выполнять одинаковый объем работы, но путешественник выполняет ее значительно быстрее, чем скалолаз.Величина, связанная со скоростью выполнения определенного объема работы, называется мощностью. Турист имеет мощность выше , чем скалолаз.

    Мощность – это скорость выполнения работы. Это соотношение работы / времени. Математически это вычисляется с использованием следующего уравнения.

    Мощность = Работа / время

    или

    P = Вт / т

    Стандартная метрическая единица измерения мощности – Вт .Как следует из уравнения мощности, единица мощности эквивалентна единице работы, деленной на единицу времени. Таким образом, ватт эквивалентен джоулям в секунду. По историческим причинам термин лошадиных сил иногда используется для описания мощности, выдаваемой машиной. Одна лошадиная сила эквивалентна примерно 750 Вт.

    Большинство машин спроектировано и построено для работы с объектами. Все машины обычно характеризуются номинальной мощностью.Номинальная мощность указывает скорость, с которой эта машина может работать с другими объектами. Таким образом, мощность машины – это соотношение работы / времени для этой конкретной машины. Автомобильный двигатель – это пример машины, которой задана номинальная мощность. Номинальная мощность относится к тому, насколько быстро автомобиль может разгонять автомобиль. Предположим, что двигатель мощностью 40 лошадиных сил может разогнать автомобиль от 0 миль / час до 60 миль / час за 16 секунд. Если бы это было так, то автомобиль с мощностью в четыре раза больше мог бы выполнять такой же объем работы за четверть времени.То есть 160-сильный двигатель мог разогнать тот же автомобиль с 0 миль / час до 60 миль / час за 4 секунды. Дело в том, что при одинаковом объеме работы мощность и время обратно пропорциональны. Уравнение мощности предполагает, что более мощный двигатель может выполнять такой же объем работы за меньшее время.

    Человек – это также машина с номинальной мощностью . Некоторые люди более властны, чем другие. То есть некоторые люди способны выполнять тот же объем работы за меньшее время или больше за то же время.Обычная физическая лаборатория включает в себя быстрый подъем по лестнице и использование информации о массе, росте и времени для определения личных способностей ученика. Несмотря на диагональное движение по лестнице, часто предполагается, что горизонтальное движение является постоянным, и вся сила от ступенек используется для подъема ученика вверх с постоянной скоростью. Таким образом, вес ученика равен силе, которая действует на ученика, а высота лестницы – это смещение вверх. Предположим, что Бен Пумпинирон поднимает свое 80-килограммовое тело на 2.0-метровый подъезд за 1,8 секунды. Если бы это было так, то мы могли бы рассчитать номинальную мощность Бена . Можно предположить, что Бен должен приложить к лестнице нисходящую силу 800 Ньютон, чтобы поднять свое тело. Поступая таким образом, лестница толкала тело Бена вверх с достаточной силой, чтобы поднять его тело вверх по лестнице. Также можно предположить, что угол между силой лестницы на Бена и смещением Бена равен 0 градусов. Используя эти два приближения, можно определить номинальную мощность Бена, как показано ниже.

    Номинальная мощность Бена – 871 Вт. Ему 9 лошадка вполне .

    Другая формула мощности

    Выражение для мощности – работа / время. А поскольку выражение для работы – это сила * смещение, выражение для мощности можно переписать как (сила * смещение) / время. Поскольку выражение для скорости – это смещение / время, выражение для мощности можно еще раз переписать как «сила * скорость».Это показано ниже.

    Это новое уравнение мощности показывает, что мощная машина одновременно сильна (большая сила) и быстра (большая скорость). Мощный автомобильный двигатель – сильный и быстрый. Мощная сельскохозяйственная техника – прочная и быстрая. Сильный тяжелоатлет силен и быстр. Сильный лайнсмен футбольной команды силен и быстр. Машина , которая достаточно сильна, чтобы приложить большую силу, чтобы вызвать смещение за небольшой промежуток времени (т.е.е., большая скорость) – машина мощная.

    Проверьте свое понимание

    Используйте свое понимание работы и власти, чтобы ответить на следующие вопросы. По завершении нажмите кнопку, чтобы просмотреть ответы.

    1. Два студента-физика, Уилл Н. Эндейбл и Бен Пумпинирон, в зале для тяжелой атлетики. Уилл поднимает 100-фунтовую штангу над головой 10 раз за одну минуту; Бен поднимает 100-фунтовую штангу над головой 10 раз за 10 секунд.Какой студент больше всего работает? ______________ Какой ученик дает больше всего энергии? ______________ Объясните свои ответы.

    2. В физической лаборатории Джек и Джилл взбежали на холм. Джек вдвое массивнее Джилл; тем не менее, Джилл преодолевает то же расстояние за половину времени. Кто работал больше всего? ______________ Кто доставил больше всего энергии? ______________ Объясните свои ответы.


    3. Уставшая белка (масса около 1 кг) отжимается, прикладывая силу, поднимающую ее центр масс на 5 см, чтобы выполнить работу всего на 0,50 Дж. Если уставшая белка проделает всю эту работу за 2 секунды, то определите ее мощность.

    4. При подтягивании вверх студентка-физик поднимает ее 42.0-кг тело на дистанцию ​​0,25 метра за 2 секунды. Какую силу развивают бицепсы ученика?

    5. Ежемесячный счет за электроэнергию в вашей семье часто выражается в киловатт-часах. Один киловатт-час – это количество энергии, доставленное потоком 1 киловатт электроэнергии за один час. Используйте коэффициенты преобразования, чтобы показать, сколько джоулей энергии вы получаете, покупая 1 киловатт-час электроэнергии.

    6. Эскалатор используется для перемещения 20 пассажиров каждую минуту с первого этажа универмага на второй. Второй этаж находится на высоте 5,20 метра над первым этажом. Средняя масса пассажира – 54,9 кг. Определите требуемую мощность эскалатора, чтобы переместить это количество пассажиров за это время.

    Формула моментума

    Формула моментума Вопросы:

    1) Общая масса мотоцикла и человека, едущего на нем, составляет 200.0 кг. Если гонщик движется с постоянной скоростью 30,0 м / с, каков импульс мотоцикла и гонщика?

    Ответ: Импульс можно найти по формуле:

    p = mv

    p = (200,0 кг) (30,0 м / с)

    p = 6000 кг · м / с

    Импульс мотоцикла и гонщика 6000 кг · м / с.

    2) Хоккейная шайба скользит по льду со скоростью 43,80 м / с. Имеет массу 0,165 кг.Шайба попадает в камень для керлинга весом 19,10 кг, который изначально находится в состоянии покоя. Хоккейная шайба отскакивает от камня и скользит в противоположном направлении со скоростью 43,0 м / с. Какова скорость скручивающейся скалы после столкновения?

    Ответ: Этот вопрос зависит от сохранения количества движения при упругих столкновениях. Общий импульс до равен общему импульсу после:

    p шайба, перед + p скала, перед = p шайба, после + p скала, после

    м шайба v шайба, до + м скала v скала, до = м шайба v шайба, после + м скала v скала, после

    Масса шайбы м шайба = 0.165 кг . Масса породы м, порода = 19,10 кг. Скорость шайбы до столкновения v шайба, до = +43,80 м / с. Скорость породы перед столкновением составляет v камень, до = 0,0 м / с, потому что она находилась в состоянии покоя. Скорость шайбы после столкновения отрицательна, потому что она двигалась в противоположном направлении, как и до столкновения: v шайба, после = -43,0 м / с.

    (0,165 кг) (43,80 м / с) + (19,10 кг) (0.0 м / с) = (0,165 кг) (- 43,0 м / с) + (19,10 кг) v порода, после

    (0,165 кг) (43,80 м / с) = – (0,165 кг) (43,0 м / с) + (19,10 кг) v скала, после

    Теперь уравнение можно изменить, чтобы найти камень v после .

    (0,165 кг) (43,80 м / с) + (0,165 кг) (43,0 м / с) = (19,10 кг) против породы , после

    v скала, после = +0,750 м / с

    Скорость керлинга после столкновения с хоккейной шайбой плотностью льда равна v рок, после = +0.750 м / с.

    Физические формулы

    Это На странице есть все необходимые вам формулы физики. В первом разделе у нас есть СИ единицы измерения. В следующем разделе мы рассмотрим уравнения механики и уравнения электричества.

    Эти Уравнения физики описывают взаимосвязь между скоростью, ускорением и силы и т. д. Как только мы поймем основную физику, уравнения могут служить в качестве рамки, которые мы можем использовать, чтобы понять и предсказать исход физического явления. Конечно, эти уравнения также будут неоценимы, когда дело доходит до расчета неизвестные значения из известных.

    Физика это наука, которая в значительной степени полагается на математические навыки. Основным из них является алгебра, поскольку вам нужно иметь возможность заменять и переставлять уравнение
    , если это необходимо. Помните, что мы всегда можем изменить формулу в соответствии с конкретным приложением.

    Примечание: Все эти физические формулы требуют использования единиц СИ
    (Международная система единиц)

    Единицы СИ

    Кол-во Количество символ Блок символ
    Масса м килограмм кг
    Усилие F Ньютон N
    расстояние d метров м
    скорость v Скорость v
    Давление с. Паскаль Па
    Работа Вт Джоулей Дж
    Энергия E Джоулей Дж
    Время т секунд с

    https: // www.easy-science-experiments.com/#show-hide

    Наши лучшие идеи подарков для молодых ученых
    https://www.easy-science-experiments.com/#show-hide

    Уравнения механики

    $$ v = \ frac {\ Delta d} {\ Delta t} $$

    $$ velocity = \ frac {\ text {изменение смещения}} {\ text {изменение во времени}} $$

    $$ a = \ frac {\ Delta v} {\ Delta t} $$

    $$ ускорение = \ frac {\ text {изменение скорости}} {\ text {изменение во времени}} $$

    $$ Сила = масса \ раз ускорение $$

    $$ Работа = Сила \ умноженное на расстояние $$

    $$ Мощность = \ frac {Работа} {время} $$

    $$ крутящий момент = Сила \ умноженное на расстояние $$

    $$ Force = – \ text {постоянная эластичности} \ times extension $$

    Уравнения кругового движения

    $$ a_c = \ frac {v ^ 2} {r} $$

    $$ \ text {центростремительное ускорение} = \ frac {скорость ^ 2} {радиус} $$

    $$ F_c = \ frac {mv ^ 2} {r} $$

    $$ \ text {центростремительная сила} = \ frac {масса \ умноженная на скорость ^ 2} {радиус} $$

    $$ C = 2 \ pi r $$

    $$ Окружность = 2 \ пи \ умножить на радиус $$

    Импульс

    $$ импульс = масса \ умноженная на скорость $$

    $$ \ Delta p = F \ Delta t $$

    $$ \ text {изменение импульса} = Сила \ раз \ text {изменение во времени} $$

    Уравнения энергии

    $$ E_p = \ frac {1} {2} kx ^ 2 $$

    $$ \ text {Упругая потенциальная энергия} = \ frac {1} {2} \ text {постоянная упругости} \ times extension ^ 2 $$

    $$ E_k = \ frac {1} {2} mv ^ 2 $$

    $$ \ text {Кинетическая энергия} = \ frac {1} {2} масса \ умноженная на скорость ^ 2 $$

    $$ \ Delta E_p = мг \ Delta ч $$

    $$ \ text {гравитационная потенциальная энергия} = масса \ times \ text {ускорение свободного падения} \ times \ text {изменение высоты} $$

    Ep означает энергетический потенциал, который представляет собой накопленную энергию.2 + 2ad $$

    $$ d = (\ frac {v_f + v_f} {2}) t $$

    Уравнения электроэнергии

    $$ Напряжение = ток \ умноженное на сопротивление $$

    $$ P = I V $$

    $$ P = \ frac {\ Delta E} {t} $$

    $$ Мощность = Ток \ раз Напряжение $$

    $$ Мощность = \ frac {Изменение энергии} {время} $$

    $$ R_T = R_1 + R_2 + R_3 + … $$

    Полное сопротивление резистора, включенного параллельно, равно сумме обратных сопротивлений резисторов.

    $$ \ frac {1} {R_T} = \ frac {1} {R_1} + \ frac {1} {R_2} + \ frac {1} {R_3} +… $$

    Суммарное сопротивление последовательно включенных резисторов – сумма всех последовательно включенных резисторов.

    $$ \ text {Напряженность электрического поля} = \ frac {\ text {Разница потенциалов (напряжение)}} {расстояние} $$

    $$ \ text {Электрическая сила} = \ text {Напряженность электрического поля} \ times charge $$

    $$ Current = \ frac {charge} {timt} $$

    $$ V = \ frac {\ Delta E} {Q} $$

    $$ V = \ frac {изменение энергии} {Charge} $$

    $$ \ Delta E = qEd $$

    $$ \ text {Изменение энергии} = заряд \ раз \ text {напряженность электрического поля} \ раз расстояние $$


    https: // www.easy-science-experiments.com/#show-hide

    Перестановка физических формул

    Помните: Мы можем переставить физические формулы, применяя простую алгебру, чтобы гарантировать, что все символы в правой части уравнения известны.
    Например, если мы знаем силу, действующую на объект, и массу объекта, как рассчитать ускорение, испытываемое объектом, с помощью уравнения:

    $$ F = ma $$

    Нам нужно переставить уравнение так, что a находится в левой части, а F – в правой.

    $$ a = \ frac {m} {F} $$

    Теперь мы можем вставить m и F, чтобы получить a. Пока есть только одно неизвестное, мы можем легко изменить уравнения, чтобы дать нам ответ.

    Вернитесь из «Физических формул» на ГЛАВНУЮ.



    https://www.easy-science-experiments.com/#show-hide

    Power (Физика): Определение, Формула, Единицы, Как найти (с примерами)

    Обновлено 28 декабря 2020 г.

    Автор: Эми Дусто

    Бодибилдер и пятиклассник могли унести все книги с полки вверх лестничный пролет, но вряд ли они справятся с задачей за то же время.Бодибилдер, вероятно, будет быстрее, потому что у нее рейтинг мощности выше , чем у пятиклассника.

    Точно так же гоночный автомобиль с высокой мощностью лошадиных сил сможет проехать дальше намного быстрее, чем лошадь.

    TL; DR (слишком долго; не читал)

    Мощность – это мера того, сколько работы выполнено за временной интервал.

    Краткое замечание о лошадиных силах: этот термин предназначен для сравнения мощности парового двигателя с мощностью лошади, поскольку двигатель мощностью 700 лошадиных сил может выполнять примерно в 700 раз больше работы, чем одна лошадь.Это восходит к тому времени, когда паровые двигатели были новыми, и один из самых выдающихся изобретателей, работавших над повышением их эффективности, Джеймс Ватт, придумал этот термин как способ убедить среднего человека в их ценности.

    Формулы для мощности

    Есть два способа рассчитать мощность в зависимости от того, какая информация доступна. Кроме того, есть две единицы мощности, которые одинаково действительны.

    1. Мощность в единицах работы и времени:

    P = \ frac {W} {t}

    Где работа Вт измеряется в Ньютон-метрах (Нм), а время т измеряется в секундах (с).

    2. Мощность в единицах силы и скорости:

    P = Fv

    Где сила F выражается в Ньютонах (Н), а скорость v выражается в метрах в секунду (м / с). .

    Эти уравнения не эквивалентны случайным образом. Второе уравнение может быть получено из первого следующим образом:

    Обратите внимание, что работа совпадает с усилие на перемещение:

    W = Fd

    Подставьте это в первое уравнение мощности:

    Затем, поскольку смещение в любую единицу времени равно скорости (v = d / t), перепишите члены в конце как v , чтобы получить второе уравнение мощности.

    Единицы мощности

    Единица мощности в системе СИ p обычно представлена ​​как Вт (Вт) , названная в честь того же Джеймса Ватта, который проектировал двигатели и сравнивал их с лошадьми. На бирках лампочек и других бытовых приборов этот блок обычно указывается.

    Однако рассмотрение второй формулы мощности приводит к другой единице. Сила, умноженная на скорость, дает измерение в единицах ньютон-метров в секунду (Нм / с). Затем, поскольку единица энергии Джоуль также определяется как один Ньютон-метр (Нм), первую часть этой энергии можно переписать как Джоуль, в результате чего получится вторая единица мощности СИ: Джоуль в секунду (Дж. / с).

    Как стать могущественным

    Рассмотрение определения силы и двух способов ее нахождения дает несколько способов увеличить силу чего-то : увеличить его силу (использовать больше силы ) или получить та же работа выполняется быстрее (уменьшение т или увеличение т ). Мощная машина – это сильная и быстрая , а слабая – ни то, ни другое. легче и быстрее может работать , более мощный объект, выполняющий работу.

    Это также означает, что очень сильный тренажер, скажем, очень мускулистый бодибилдер, может все еще испытывать недостаток в мощности . Человек, который может поднять очень тяжелый груз, но только очень медленно, менее силен, чем тот, кто может поднять его быстро.

    Точно так же очень быстрая машина или человек, который мало что делает, кто-то быстро крутится на месте, но ни к чему не приходит, на самом деле не является мощным.

    Пример расчета мощности

    1. Усэйн Болт выработал мощность около 25 Вт в своем рекордном спринте на 100 м, который занял 9.58 секунд. Сколько работы он проделал?

    Поскольку указаны P и t , а W неизвестно, используйте первое уравнение:

    P = \ frac {W} {t} \ подразумевает 25 = \ frac { W} {9.58} \ подразумевает W = 239.5 \ text {Nm}

    2. С какой средней силой он давил на землю во время бега?

    Так как работа в Нм уже известна, как и рабочий объем в метрах, деление на длину гонки даст силы (другими словами, работа то же самое, что сила, умноженная на смещение: W = F × d):

    \ frac {239.5} {100} = 2.395 \ text {N}

    3. Какую мощность вырабатывает человек весом 48 кг, который тратит 6 секунд на подъем по 3-метровой лестнице?

    В этой задаче указаны смещение и время, что позволяет быстро вычислить скорость:

    v = \ frac {d} {t} = \ frac {3} {6} = 0,5 \ text {м / с}

    Второе уравнение мощности учитывает скорость, но также включает силу. Человек, поднимающийся по лестнице, пытается противостоять силе тяжести. Итак, силу в этом случае можно найти, используя их массу и ускорение свободного падения, которое на Земле всегда равно 9.8 м / с 2 .

    F_ {grav} = mg = 48 \ times 9,8 = 470,4 \ text {N}

    Теперь сила и скорость укладываются во вторую формулу мощности:

    = Fv = 470,4 \ times 0,5 = 235,2 \ text {J / s}

    Обратите внимание, что решение оставить здесь единицы измерения Дж / с, а не ватты, является произвольным. Столь же приемлемый ответ – 235,2 Вт.

    4. Одна лошадиная сила в единицах СИ составляет около 746 Вт, что основано на нагрузке, которую пригодная лошадь могла бы выдержать в течение одной минуты. Сколько работы проделала лошадь-пример за это время?

    Единственный шаг перед включением значений мощности и времени в первое уравнение – убедиться, что время указано в правильных единицах СИ, в секундах, путем переписывания одной минуты на 60 секунд.Тогда:

    P = \ frac {W} {t} \ подразумевает 746 = \ frac {W} {60} \ implies W = 44,670 \ text {Nm}

    Киловатт и электричество

    Многие коммунальные предприятия взимают плату с клиентов. плата основана на использовании киловатт-часов и киловатт-часов. Чтобы понять значение этой общей единицы электроэнергии, начните с разбивки единиц.

    Префикс килограмм означает 1000, поэтому киловатт (кВт) равен 1000 ватт. Таким образом, киловатт-час (кВтч) – это количество киловатт, используемое за один час времени.

    Для подсчета киловатт-часов умножьте количество киловатт на использованные часы. Таким образом, если кто-то использует 100-ваттную лампочку в течение 10 часов, он в общей сложности израсходует 1000 ватт-часов или 1 кВт-ч электроэнергии.

    Киловатт-час Примеры проблем

    1. Электроэнергетика взимает 0,12 доллара за киловатт-час. Очень мощный вакуум 3000 Вт используется в течение 30 минут. Сколько стоит это количество энергии домовладельцам?

    3 \ text {кВт} \ times 0.5 \ text {h} = 1,5 \ text {кВтч} \ text {и} 1,5 \ text {кВтч} \ times 0,12 \ text {долларов / кВтч} = \ 0,18 доллара США

    2. Та же коммунальная компания кредитует домашнему хозяйству 10 долларов на каждые 4 кВтч электроэнергии возвращается в сеть. Солнце дает около 1000 Вт мощности на квадратный метр. Если солнечный элемент площадью два квадратных метра в доме собирает энергию в течение 8 часов, сколько денег он приносит?

    Учитывая информацию в задаче, солнечный элемент должен быть способен собирать 2 000 Вт от Солнца или 2 кВт. За 8 часов это 16 кВтч.

    \ frac {\ $ 10} {4 \ text {kWh}} \ times 16 \ text {kWh} = \ $ 40

    Физический алфавит …

    килограмм

    Нижний регистр букв

    Верхний case Letters

    Греческие буквы * и их имена Сокращенное обозначение уравнения для физической величины Символ единицы С.I. Префикс и его значение
    NB оно всегда предшествует символу единицы
    а А α Α альфа

    А = площадь

    A = нуклон число (атомная масса)

    а = ускорение

    а = Константа Вейна

    а = альфа-частица

    а = атто х 10 -18
    б B β Β бета

    В = плотность магнитного потока

    б = бета-частица

    В = звонок (сила звука)

    Бк = беккерель (активность)

    c C χ Χ χ

    С = емкость

    c = скорость света

    c = удельная теплоемкость

    o С = градус Цельсия (температура) с = сенти х 10 -2
    d D δ Δ δ

    д = диаметр

    d = расстояние

    D = расстояние от экрана с рисунком бахромы

    D = поглощенная доза

    Δ = изменение в

    δ = небольшое изменение

    D = диоптрия (оптическая сила линзы)

    дБ = децибел (интенсивность звука)

    д = деци

    да = дека (или дека)

    х 10 -1

    х 10 1

    е E ε Ε ε

    e = заряд электрона

    E = энергия

    E k = кинетическая энергия

    E = напряженность электрического поля

    E = модуль Юнга

    ε = ЭДС

    ε = деформация растяжения

    ε o = диэлектрическая проницаемость свободного пространства

    эВ = электрон-вольт (энергия) E = exa х 10 18
    ж F φ Φ фи

    F = усилие

    f = частота

    f = фокусное расстояние

    f e = фокусное расстояние линзы окуляра

    f o = фокусное расстояние линзы объектива

    Φ = поток

    φ = работа выхода

    F = фарад (емкость) f = femto х 10 -15
    г г γ Γ гамма

    г = напряженность гравитационного поля

    g = ускорение свободного падения

    G = гравитационная постоянная

    G = проводимость

    γ = гамма-луч

    Гр = серый (поглощенная доза) G = гига х 10 9
    час ЧАС η Η эта

    h = высота

    h = постоянная Планка

    H = эквивалент дозы>

    H = постоянная Хаббла

    Η = коэффициент вязкости

    H = Генри (индуктивность)

    Гц = герц (частота)

    ч = гектар х 10 2
    я я ι Ι йота

    I = текущий

    I 0 = пиковый ток

    I = интенсивность звука

    I = момент инерции

    j J θ Θ тета

    Дж = плотность тока

    Дж = момент инерции

    Θ = угол

    Дж = джоуль (энергия)
    k K κ Κ каппа

    k = постоянная Больцмана

    k = жесткость пружины

    K = кельвин (абсолютная температура)

    кг = килограмм (масса)

    k = х 10 3
    л L λ Λ лямбда

    л = длина

    л = скрытая теплоемкость

    λ = длина волны

    λ = постоянная затухания

    L = собственная индуктивность

    L = угловой момент

    ln = натуральный логарифм

    журнал = журнал по основанию 10

    л = литр (= 1000 см 3 ) – измерение объема
    м M м μ му

    m = масса

    M = увеличение

    μ = проницаемость

    μ = коэффициент трения

    м = метр (длина)

    м 2 = квадратный метр (площадь)

    м 3 = кубический метр (объем)

    M = мега

    м =

    милли

    мк = микро

    х 10 6

    х 10 -3

    х 10 -6

    п N ν Ν nu

    N = число

    N O = оригинальный номер

    N A = постоянная Авогадро

    N = количество витков провода

    n = количество родинок

    n = порядок дифракции

    n = количество носителей заряда на единицу объема

    n = показатель преломления

    N = ньютон (сила или вес) п = нано х 10 -9
    о О ο Ο омикрон НЕ используется ни для чего – его слишком легко спутать с числом ноль
    п п π Π пи

    P = мощность

    p = давление

    p = импульс

    π = 3.14

    Па = паскаль (давление)

    p = pico

    P = пета

    х 10 -12

    х 10 15

    q Q

    Q = заряд

    Q = тепло энергия

    Q = качество фактор

    р р ρ Ρ rho

    r = радиус

    R = сопротивление

    R = молярная газовая постоянная

    R = реакционная сила

    ρ = плотность

    ρ = удельное сопротивление

    рад = радиан

    s S σ Σ сигма

    s = смещение (векторная версия расстояния)

    s = ширина щели

    σ = проводимость

    σ = растягивающее напряжение

    σ = постоянная Стефана

    Σ = сумма

    с = секунда (время)

    Зв = зиверт (эквивалент дозы)

    S = сименс (проводимость)

    т Т τ Τ тау

    t = время

    T = температура

    T = период формы волны

    T 1/2 = период полураспада

    T E = эффективный период полураспада

    T B = биогенный период полураспада

    T P = физический период полураспада

    T = тесла (плотность магнитного потока) Т = тера х 10 12
    ты U υ Υ ипсилон

    u = начальная скорость

    u = расстояние изображения

    U = U-значение

    U = внутреннее тепло системы

    u = атомная единица массы (масса на атомных уровнях)
    v V

    v = скорость

    v = конечная скорость (при использовании вместе с ‘u’)

    v = расстояние до изображения

    V = объем

    В = разность потенциалов

    В 0 = пиковое напряжение

    В = вольт (электрический потенциал)
    ш W ω Ω омега

    Вт = выполненная работа

    ω = угловая скорость

    w = ширина бахромы

    Ом = Ом (электрическое сопротивление)

    Вт = ватт (мощность)

    Wb = weber (магнитный поток)

    Икс Икс χ Χ чи

    x = ширина

    X = реактивное сопротивление

    у Y ξ Ξ xi

    y = высота

    г = йокто

    Y = йота

    х 10 -24

    х 10 24

    z Z ζ Ζ дзета

    z = глубина

    Z = протонное число (атомный номер)

    z = zepto

    Z = дзета

    х 10 -21

    х 10 21

    ПРИМЕЧАНИЕ НАСКОЛЬКО ВАЖНА ПЕРЕДАЧА БУКВЫ – УЗНАТЬ ПРИНЯТЫЕ СИМВОЛЫ ОСТОРОЖНО!!!

    NB A кандидаты уровня должны:


    (а) определяют символы, используемые в уравнении и
    (b) условия, при которых уравнения применяются !!!

    8.1 Линейный импульс, сила и импульс – Физика

    Цели обучения секции

    К концу этого раздела вы сможете делать следующее:

    • Опишите импульс, то, что может изменить импульс, импульс и теорему об импульсе-импульсе
    • Опишите второй закон Ньютона с точки зрения количества движения
    • Решение задач с помощью теоремы об импульсе-импульсе

    Поддержка учителей

    Поддержка учителей

    Цели обучения в этом разделе помогут вашим ученикам овладеть следующими стандартами:

    • (6) Научные концепции.Учащийся знает, что в физической системе происходят изменения, и применяет законы сохранения энергии и количества движения. Ожидается, что студент:
      • (C) вычисляет механическую энергию, мощность, генерируемую внутри, импульс, приложенный к, и импульс физической системы.

    Раздел Ключевые термины

    изменение импульса импульс теорема об импульсе-импульсе Импульс

    Поддержка учителей

    Поддержка учителей

    [BL] [OL] Просмотрите инерцию и законы движения Ньютона.

    [AL] Начать обсуждение движения и столкновения. На примере футболистов укажите, что и масса, и скорость объекта являются важными факторами при определении воздействия столкновений. Направление, а также величина скорости очень важны.

    Импульс, импульс и теорема импульс-импульс

    Линейный импульс – это произведение массы системы на ее скорость. В форме уравнения импульс движения p равен

    .

    Из уравнения видно, что импульс прямо пропорционален массе объекта ( м ) и скорости ( v ).Следовательно, чем больше масса объекта или чем больше его скорость, тем больше его импульс. Большой, быстро движущийся объект имеет больший импульс, чем более мелкий и медленный объект.

    Импульс является вектором и имеет то же направление, что и скорость v . Поскольку масса является скаляром, когда скорость находится в отрицательном направлении (т. Е. Противоположно направлению движения), импульс также будет в отрицательном направлении; и когда скорость в положительном направлении, импульс также будет в положительном направлении.Единица измерения количества движения в системе СИ – кг м / с.

    Импульс настолько важен для понимания движения, что физики, такие как Ньютон, назвали его величиной движения . Сила влияет на импульс, и мы можем изменить второй закон движения Ньютона, чтобы показать взаимосвязь между силой и импульсом.

    Вспомните наше исследование второго закона движения Ньютона ( F net = м a ). Ньютон фактически сформулировал свой второй закон движения в терминах количества движения: чистая внешняя сила равна изменению количества движения системы, деленному на время, в течение которого он изменяется.Изменение импульса – это разница между конечным и начальным значениями импульса.

    В форме уравнения этот закон имеет вид

    , где F net – чистая внешняя сила, ΔpΔp – изменение количества движения, а ΔtΔt – изменение во времени.

    Мы можем найти ΔpΔp, переписав уравнение

    будет

    FnetΔtFnetΔt известен как импульс, и это уравнение известно как теорема импульс-импульс. Из уравнения мы видим, что импульс равен средней чистой внешней силе, умноженной на время ее действия.Это равно изменению импульса. Действие силы на объект зависит от того, как долго оно действует, а также от силы силы. Импульс – полезное понятие, поскольку оно позволяет количественно оценить влияние силы. Очень большая сила, действующая в течение короткого времени, может иметь большое влияние на импульс объекта, например, сила удара ракетки по теннисному мячу. Небольшая сила может вызвать такое же изменение импульса, но действовать ей придется гораздо дольше.

    Поддержка учителя

    Поддержка учителя

    [OL] [AL] Объясните: большой, быстро движущийся объект имеет больший импульс, чем меньший и более медленный объект.Это качество называется импульсом.

    [BL] [OL] Просмотрите уравнение второго закона движения Ньютона. Укажите на два разных уравнения закона.

    Второй закон Ньютона в терминах количества движения

    Когда второй закон Ньютона выражается в единицах количества движения, его можно использовать для решения задач, в которых масса изменяется, поскольку Δp = Δ (mv) Δp = Δ (mv). В более традиционной форме закона, с которым вы привыкли работать, масса считается постоянной. Фактически, эта традиционная форма является частным случаем закона, в котором масса постоянна.Fnet = maFnet = ma фактически выводится из уравнения:

    Чтобы понять взаимосвязь между вторым законом Ньютона в его двух формах, давайте воссоздадим вывод Fnet = maFnet = ma из

    .

    , заменив определения ускорения и импульса.

    Изменение количества движения ΔpΔp равно

    Если масса системы постоянна, то

    При замене ΔpΔp mΔvmΔv второй закон движения Ньютона становится

    Fnet = ΔpΔt = mΔvΔtFnet = ΔpΔt = mΔvΔt

    для постоянной массы.

    Потому что

    мы можем заменить, чтобы получить знакомое уравнение

    , когда масса системы постоянна.

    Поддержка учителей

    Поддержка учителей

    [BL] [OL] [AL] Покажите две различные формы второго закона Ньютона и то, как одну можно вывести из другой.

    Советы для успеха

    Мы только что показали, как Fnet = maFnet = ma применяется только тогда, когда масса системы постоянна. Примером, когда эта формула неприменима, может быть движущаяся ракета, которая сжигает достаточно топлива, чтобы значительно изменить массу ракеты.В этом случае вам нужно будет использовать второй закон Ньютона, выраженный в единицах количества движения, чтобы учесть изменение массы.

    Snap Lab

    Движение рук и импульс

    В этом упражнении вы будете экспериментировать с различными типами движений рук, чтобы получить интуитивное понимание взаимосвязи между силой, временем и импульсом.

    • один мяч
    • одна ванна с водой

    Процедура:

    1. Попробуйте поймать мяч, пока дает мячом, подтягивая руки к телу.
    2. Затем попробуйте поймать мяч, не двигая руками.
    3. Попадать в ванну полной ладонью. Ваша полная ладонь изображает пловца, который делает шлепок животом.
    4. После того, как вода осядет, снова ударьте по воде, погрузив руку в воду сначала пальцами. Ваша рука для ныряния изображает ныряющего пловца.
    5. Объясните, что происходит в каждом случае и почему.

    Проверка захвата

    Какие еще примеры движений, на которые влияет импульс?

    1. столкновение футболиста с другим игроком или автомобиль, движущийся с постоянной скоростью
    2. Автомобиль, движущийся с постоянной скоростью, или объект, движущийся снарядом
    3. Автомобиль, движущийся с постоянной скоростью, или ракетка, ударяющая по мячу
    4. столкновение футболиста с другим игроком или удар ракеткой по мячу

    Поддержка учителя

    Поддержка учителя

    [OL] [AL] Обсудите удар, который ощущается при падении или прыжке.Перечислите факторы, влияющие на это влияние.

    Ссылки на физику

    Инженерное дело: спасение жизней с помощью концепции импульса

    Автомобили за последние несколько десятилетий стали намного безопаснее. Ремни безопасности играют важную роль в обеспечении безопасности автомобиля, предотвращая попадание людей в лобовое стекло в случае аварии. Другие функции безопасности, такие как подушки безопасности, менее заметны или очевидны, но также эффективны для снижения смертельных исходов при автокатастрофах (см. Рисунок 8.2). Многие из этих функций безопасности используют концепцию импульса из физики.Напомним, что импульс – это чистая сила, умноженная на продолжительность удара. Математически это выражалось как Δp = FnetΔtΔp = FnetΔt.

    Рис. 8.2 На автомобилях установлены такие устройства безопасности, как подушки безопасности и ремни безопасности.

    Подушки безопасности

    позволяют пассажирам в автомобиле действовать в течение более длительного времени при внезапной остановке. Изменение импульса для пассажира одинаково независимо от того, сработала подушка безопасности или нет. Но сила, заставляющая человека остановиться, будет намного меньше, если она будет действовать в течение большего времени.Изменив уравнение для импульса, чтобы найти силу Fnet = ΔpΔt, Fnet = ΔpΔt, вы можете увидеть, как увеличение ΔtΔt при неизменном ΔpΔp приведет к уменьшению F net . Это еще один пример обратной зависимости. Точно так же мягкая приборная панель увеличивает время действия силы удара, тем самым уменьшая силу удара.

    Сегодня автомобили состоят из множества пластиковых компонентов. Одним из преимуществ пластмасс является их меньший вес, что приводит к лучшему расходу топлива.Еще одно преимущество заключается в том, что автомобиль может смяться при столкновении, особенно в случае лобового столкновения. Более длительное время столкновения означает, что сила, действующая на пассажиров в автомобиле, будет меньше. Смертность во время автомобильных гонок резко снизилась, когда жесткие рамы гоночных автомобилей были заменены деталями, которые могли смяться или разрушиться в случае аварии.

    Проверка захвата

    Возможно, вы слышали совет сгибать колени при прыжках. В этом примере друг предлагает вам спрыгнуть со скамейки в парке на землю, не сгибая колен.Вы, конечно, отказываетесь. Объясните другу, почему это было бы глупо. Покажите это с помощью теоремы об импульсе-импульсе.

    1. Сгибание колен увеличивает время удара, тем самым уменьшая силу.
    2. Сгибание колен сокращает время удара, тем самым уменьшая силу.
    3. Сгибание колен увеличивает время удара, увеличивая, таким образом, силу.
    4. Сгибание колен сокращает время удара, увеличивая, таким образом, силу.

    Решение задач с использованием теоремы об импульсе-импульсе

    Поддержка учителей

    Поддержка учителей

    Расскажите о различных стратегиях, которые следует использовать при решении задач. Убедитесь, что учащиеся знают допущения, сделанные в каждом уравнении относительно того, что определенные величины являются постоянными или некоторые величины пренебрежимо малы.

    Рабочий пример

    Расчет импульса: футболист и футбол

    (a) Рассчитайте импульс футболиста массой 110 кг, бегущего со скоростью 8 м / с.(b) Сравните импульс игрока с импульсом футбольного мяча весом 0,410 кг, брошенного со скоростью 25 м / с.

    Стратегия

    Нет информации о направлении футболиста или футбольного мяча, поэтому мы можем вычислить только величину импульса, p . (Курсивом обозначена величина.) В обеих частях этого примера величина импульса может быть вычислена непосредственно из определения импульса:

    Решение для (а)

    Чтобы найти импульс игрока, подставьте известные значения массы и скорости игрока в уравнение.

    pplayer = (110 кг) (8 м / с) = 880 кг⋅м / сpplayer = (110 кг) (8 м / с) = 880 кг⋅м / с

    Решение для (b)

    Чтобы найти импульс мяча, подставьте известные значения массы и скорости мяча в уравнение.

    мяч = (0,410 кг) (25 м / с) = 10,25 кг⋅м / мяч = (0,410 кг) (25 м / с) = 10,25 кг⋅м / с

    Отношение количества движения игрока к импульсу мяча составляет

    pplayerpball = 88010.3 = 85.9.pplayerpball = 88010.3 = 85.9.

    Обсуждение

    Хотя мяч имеет большую скорость, игрок имеет гораздо большую массу.Следовательно, импульс игрока примерно в 86 раз больше, чем импульс футбола.

    Рабочий пример

    Расчетное усилие: ракетка Винус Уильямс

    Во время Открытого чемпионата Франции 2007 года Винус Уильямс (рис. 8.3) показала самую быструю подачу в матче лучших женщин, достигнув скорости 58 м / с (209 км / ч). Какая средняя сила была приложена к теннисному мячу весом 0,057 кг ракеткой Williams? Предположим, что скорость мяча сразу после удара составляла 58 м / с, горизонтальная скорость до удара незначительна и что мяч оставался в контакте с ракеткой в ​​течение 5 мс (миллисекунд).

    Рис. 8.3. Винус Уильямс играет на US Open 2013 (Эдвин Мартинес, Flickr)

    Стратегия

    Напомним, что второй закон Ньютона, выраженный в единицах количества движения, равен

    .

    Как отмечалось выше, когда масса постоянна, изменение количества движения определяется как

    Δp = mΔv = m (vf − vi), Δp = mΔv = m (vf − vi),

    , где v f – конечная скорость, а v i – начальная скорость. В этом примере даны скорость сразу после удара и изменение во времени, поэтому после решения для ΔpΔp мы можем использовать Fnet = ΔpΔtFnet = ΔpΔt, чтобы найти силу.

    Решение

    Чтобы определить изменение количества движения, подставьте значения массы, начальной и конечной скоростей в приведенное выше уравнение.

    Δp = m (vf − vi) = (0,057 кг) (58 м / с – 0 м / с) = 3,306 кг · м / с ≈ 3,3 кг · м / с Δp = m (vf − vi) = (0,057 кг) (58 м / с – 0 м / с) = 3,306 кг · м / с ≈ 3,3 кг · м / с

    8,1

    Теперь мы можем найти величину чистой внешней силы, используя Fnet = ΔpΔtFnet = ΔpΔt

    Fnet = ΔpΔt = 3,3065 × 10−3 = 661 N≈660 Н.Fnet = ΔpΔt = 3,3065 × 10−3 = 661 N≈660 Н.

    8,2

    Обсуждение

    Эта величина была средней силой, прилагаемой ракеткой Винус Уильямс к теннисному мячу во время его кратковременного удара. Эту проблему также можно решить, сначала найдя ускорение, а затем используя F net = м a , но нам пришлось бы сделать еще один шаг. В этом случае использование импульса было сокращением.

    Практические задачи

    1.

    Каков импульс шара для боулинга с массой 5 ​​\, \ text {кг} и скоростью 10 \, \ text {м / с}?

    1. 0.5 \, \ text {кг} \ cdot \ text {м / с}
    2. 2 \, \ text {kg} \ cdot \ text {m / s}
    3. 15 \, \ text {kg} \ cdot \ text {m / s}
    4. 50 \, \ text {kg} \ cdot \ text {m / s}
    2.

    Каким будет изменение количества движения, вызванное чистой силой 120 \, \ text {N}, действующей на объект в течение 2 секунд?

    1. 60 \, \ text {kg} \ cdot \ text {m / s}
    2. 118 \, \ text {kg} \ cdot \ text {m / s}
    3. 122 \, \ text {kg} \ cdot \ text {m / s}
    4. 240 \, \ text {kg} \ cdot \ text {m / s}

    Проверьте свое понимание

    3.

    Что такое импульс?

    1. сумма массы системы и ее скорости
    2. отношение массы системы к ее скорости
    3. произведение массы системы на ее скорость
    4. произведение момента инерции системы на ее скорость
    4.

    Если масса объекта постоянна, каков его импульс?

    1. Его скорость
    2. Его вес
    3. Его водоизмещение
    4. Его момент инерции
    5.

    Каково уравнение второго закона движения Ньютона в терминах массы, скорости и времени, когда масса системы постоянна?

    1. Fnet = ΔvΔmΔtFnet = ΔvΔmΔt
    2. Fnet = mΔtΔvFnet = mΔtΔv
    3. Fnet = mΔvΔtFnet = mΔvΔt
    4. Fnet = ΔmΔvΔtFnet = ΔmΔvΔt
    6.

    Приведите пример системы, масса которой непостоянна.

    1. Волчок
    2. Бейсбольный мяч, летящий по воздуху
    3. Ракета, запущенная с Земли
    4. Блок, скользящий по наклонной плоскости без трения

    Поддержка учителей

    Поддержка учителей

    Используйте вопросы «Проверьте свое понимание», чтобы оценить, усвоили ли учащиеся учебные цели этого раздела.

    Добавить комментарий

    Ваш адрес email не будет опубликован. Обязательные поля помечены *